lie-algebras-and-their-representations

Source code for my notes on representations of semisimple Lie algebras and Olivier Mathieu's classification of simple weight modules

simple-weight.tex (81392B)

   1 \chapter{Simple Weight Modules}\label{ch:mathieu}
   2 
   3 In this chapter we will expand our results on finite-dimensional simple modules
   4 of semisimple Lie algebras by considering \emph{infinite-dimensional}
   5 \(\mathfrak{g}\)-modules, which introduces numerous complications to our
   6 analysis.
   7 
   8 For instance, in the infinite-dimensional setting we can no longer take
   9 complete-reducibility for granted. Indeed, we have seen that even if
  10 \(\mathfrak{g}\) is a semisimple Lie algebra, there are infinite-dimensional
  11 \(\mathfrak{g}\)-modules which are not semisimple. For a counterexample look no
  12 further than Example~\ref{ex:regular-mod-is-not-semisimple}: the regular
  13 \(\mathfrak{g}\)-module \(\mathcal{U}(\mathfrak{g})\) is never semisimple.
  14 Nevertheless, for simplicity -- or shall we say \emph{semisimplicity} -- we
  15 will focus exclusively on \emph{semisimple} \(\mathfrak{g}\)-modules. Our
  16 strategy is, once again, that of classifying simple modules. The regular
  17 \(\mathfrak{g}\)-module hides further unpleasant surprises, however: recall
  18 from Example~\ref{ex:regular-mod-is-not-weight-mod} that
  19 \[
  20   \bigoplus_\lambda \mathcal{U}(\mathfrak{g})_\lambda
  21   = 0
  22   \subsetneq \mathcal{U}(\mathfrak{g})
  23 \]
  24 and the weight space decomposition fails for \(\mathcal{U}(\mathfrak{g})\).
  25 
  26 Indeed, our proof of the weight space decomposition in the finite-dimensional
  27 case relied heavily in the simultaneous diagonalization of commuting operators
  28 in a finite-dimensional space. Even if we restrict ourselves to simple modules,
  29 there is still a diverse spectrum of counterexamples to
  30 Corollary~\ref{thm:finite-dim-is-weight-mod} in the infinite-dimensional
  31 setting. For instance, any \(\mathfrak{g}\)-module \(M\) whose restriction to
  32 \(\mathfrak{h}\) is a free module satisfies \(M_\lambda = 0\) for all
  33 \(\lambda\) as in Example~\ref{ex:regular-mod-is-not-weight-mod}. These are
  34 called \emph{\(\mathfrak{h}\)-free \(\mathfrak{g}\)-modules}, and rank \(1\)
  35 simple \(\mathfrak{h}\)-free \(\mathfrak{sp}_{2 n}(K)\)-modules where first
  36 classified by Nilsson in \cite{nilsson}. Dimitar's construction of the so
  37 called \emph{exponential tensor \(\mathfrak{sl}_n(K)\)-modules} in
  38 \cite{dimitar-exp} is also an interesting source of counterexamples.
  39 
  40 Since the weight space decomposition was perhaps the single most instrumental
  41 ingredient of our previous analysis, it is only natural to restrict ourselves
  42 to the case it holds. This brings us to the following definition.
  43 
  44 \begin{definition}\label{def:weight-mod}\index{\(\mathfrak{g}\)-module!weight modules}\index{weights!weight modules}\index{\(\mathfrak{g}\)-module!(essential) support}
  45   A \(\mathfrak{g}\)-module \(M\) is called a \emph{weight
  46   \(\mathfrak{g}\)-module} if \(M = \bigoplus_{\lambda \in \mathfrak{h}^*}
  47   M_\lambda\) and \(\dim M_\lambda < \infty\) for all \(\lambda \in
  48   \mathfrak{h}^*\). The \emph{support of \(M\)} is the set
  49   \(\operatorname{supp} M = \{\lambda \in \mathfrak{h}^* : M_\lambda \ne 0\}\).
  50 \end{definition}
  51 
  52 \begin{example}
  53   Corollary~\ref{thm:finite-dim-is-weight-mod} is equivalent to the fact that
  54   every finite-dimensional module of a semisimple Lie algebra is a weight
  55   module. More generally, every finite-dimensional simple module of a reductive
  56   Lie algebra is a weight module.
  57 \end{example}
  58 
  59 \begin{example}\label{ex:reductive-alg-equivalence}
  60   We have seen that every finite-dimensional \(\mathfrak{g}\)-module is a
  61   weight module for semisimple \(\mathfrak{g}\). In particular, if
  62   \(\mathfrak{g}\) is finite-dimensional then the adjoint
  63   \(\mathfrak{g}\)-module \(\mathfrak{g}\) is a weight module. More generally,
  64   a finite-dimensional Lie algebra \(\mathfrak{g}\) is reductive if, and only
  65   if the adjoint \(\mathfrak{g}\)-module \(\mathfrak{g}\) is a weight module,
  66   in which case its weight spaces are given by the root spaces of
  67   \(\mathfrak{g}\)
  68 \end{example}
  69 
  70 \begin{example}
  71   Proposition~\ref{thm:high-weight-mod-is-weight-mod} is equivalent to the fact
  72   that any highest weight \(\mathfrak{g}\)-module \(M\) of highest weight
  73   \(\lambda\) is a weight module whose support is contained in \(\lambda +
  74   \mathbb{N} \Delta^- = \{\lambda - k_n \alpha_1 - \cdots - k_n \alpha_n :
  75   \alpha_i \in \Delta^+, k_i \in \mathbb{Z}, k_i \ge 0\}\). In particular,
  76   Verma modules are weight modules.
  77 \end{example}
  78 
  79 \begin{example}\label{ex:submod-is-weight-mod}
  80   Proposition~\ref{thm:max-verma-submod-is-weight} implies that the unique
  81   maximal submodule \(N(\lambda)\) of \(M(\lambda)\) is a weight module. In
  82   fact, the proof of Proposition~\ref{thm:max-verma-submod-is-weight} can be
  83   generalized to show that every submodule \(N \subset M\) of a weight module
  84   \(M\) is a weight module, and \(N_\lambda = M_\lambda \cap N\) for all
  85   \(\lambda \in \mathfrak{h}^*\).
  86 \end{example}
  87 
  88 \begin{example}\label{ex:quotient-is-weight-mod}
  89   Given a weight module \(M\), a submodule \(N \subset M\) and \(\lambda \in
  90   \mathfrak{h}^*\), it is clear that \(\mfrac{M_\lambda}{N} \subset
  91   \left(\mfrac{M}{N}\right)_\lambda\). In addition, \(\mfrac{M}{N} =
  92   \bigoplus_{\lambda \in \mathfrak{h}^*} \mfrac{M_\lambda}{N}\). Hence
  93   \(\mfrac{M}{N}\) is weight \(\mathfrak{g}\)-module with
  94   \(\left(\mfrac{M}{N}\right)_\lambda = \mfrac{M_\lambda}{N} \cong
  95   \mfrac{M_\lambda}{N_\lambda}\).
  96 \end{example}
  97 
  98 \begin{example}\label{ex:tensor-prod-of-weight-is-weight}
  99   Let \(\mathfrak{g}_1\) and \(\mathfrak{g}_2\) be Lie algebras, \(M_1\) be a
 100   weight \(\mathfrak{g}_1\)-module and \(M_2\) a weight
 101   \(\mathfrak{g}_2\)-module. Recall from Example~\ref{ex:cartan-direct-sum}
 102   that if \(\mathfrak{h}_i \subset \mathfrak{g}_i\) are Cartan subalgebras then
 103   \(\mathfrak{h} = \mathfrak{h}_1 \oplus \mathfrak{h}_2\) is a Cartan
 104   subalgebra of \(\mathfrak{g} = \mathfrak{g}_1 \oplus \mathfrak{g}_2\) with
 105   \(\mathfrak{h}^* = \mathfrak{h}_1^* \oplus \mathfrak{h}_2^*\). In this
 106   setting, one can readily check that \(M_1 \otimes M_2\) is a weight
 107   \(\mathfrak{g}\)-module with
 108   \[
 109     (M_1 \otimes M_2)_{\lambda_1 + \lambda_2}
 110     = (M_1)_{\lambda_1} \otimes (M_2)_{\lambda_2}
 111   \]
 112   for all \(\lambda_i \in \mathfrak{h}_i^*\) and \(\operatorname{supp} (M_1
 113   \otimes M_2) = \operatorname{supp} M_1 \oplus \operatorname{supp} M_2 = \{
 114   \lambda_1 + \lambda_2 : \lambda_i \in \operatorname{supp} M_i \subset
 115   \mathfrak{h}_i^*\}\).
 116 \end{example}
 117 
 118 \begin{example}\label{thm:simple-weight-mod-is-tensor-prod}
 119   Let \(\mathfrak{g} = \mathfrak{z} \oplus \mathfrak{s}_1 \oplus \cdots \oplus
 120   \mathfrak{s}_r\) be a reductive Lie algebra, where \(\mathfrak{z}\) is the
 121   center of \(\mathfrak{g}\) and \(\mathfrak{s}_1, \ldots, \mathfrak{s}_r\) are
 122   its simple components. As in
 123   Example~\ref{ex:all-simple-reps-are-tensor-prod}, any simple weight
 124   \(\mathfrak{g}\)-module \(M\) can be decomposed as
 125   \[
 126     M \cong Z \otimes M_1 \otimes \cdots \otimes M_r
 127   \]
 128   where \(Z\) is a \(1\)-dimensional representation of \(\mathfrak{z}\) and
 129   \(M_i\) is a simple weight \(\mathfrak{s}_i\)-module. The modules \(Z\) and
 130   \(M_i\) are uniquely determined up to isomorphism.
 131 \end{example}
 132 
 133 \begin{example}\label{ex:adjoint-action-in-universal-enveloping-is-weight}
 134   We would like to show that the requirement of finite-dimensionality in
 135   Definition~\ref{def:weight-mod} is not redundant. Let \(\mathfrak{g}\) be a
 136   finite-dimensional reductive Lie algebra and consider the adjoint
 137   \(\mathfrak{g}\)-module \(\mathcal{U}(\mathfrak{g})\) -- where \(X \in
 138   \mathfrak{g}\) acts by taking commutators. Given \(\alpha \in Q\), a simple
 139   computation shows \(K \langle X_1 \cdots X_n H_1 \cdots H_m : X_i \in
 140   \mathfrak{g}_{\alpha_i}, H_i \in \mathfrak{h}, \alpha_i \in \Delta, \alpha =
 141   \alpha_1 + \cdots + \alpha_n \rangle \subset
 142   \mathcal{U}(\mathfrak{g})_\alpha\). The PBW Theorem and
 143   Example~\ref{ex:reductive-alg-equivalence} thus imply that
 144   \(\mathcal{U}(\mathfrak{g}) = \bigoplus_{\alpha \in Q}
 145   \mathcal{U}(\mathfrak{g})_\alpha\) where \(\mathcal{U}(\mathfrak{g})_\alpha =
 146   K \langle X_1 \cdots X_n H_1 \cdots H_m : X_i \in \mathfrak{g}_{\alpha_i},
 147   H_i \in \mathfrak{h}, \alpha_i \in \Delta, \alpha = \alpha_1 + \cdots +
 148   \alpha_n \rangle\). However, \(\dim \mathcal{U}(\mathfrak{g})_\alpha =
 149   \infty\). For instance, \(\mathcal{U}(\mathfrak{g})_0\) is \emph{precisely}
 150   the commutator of \(\mathfrak{h}\) in \(\mathcal{U}(\mathfrak{g})\), which
 151   contains \(\mathcal{U}(\mathfrak{h})\) and is therefore infinite-dimensional.
 152 \end{example}
 153 
 154 \begin{note}
 155   We should stress that the weight spaces \(M_\lambda \subset M\) of a given
 156   weight \(\mathfrak{g}\)-module \(M\) are \emph{not}
 157   \(\mathfrak{g}\)-submodules. Nevertheless, \(M_\lambda\) is a
 158   \(\mathfrak{h}\)-submodule. More generally, \(M_\lambda\) is a
 159   \(\mathcal{U}(\mathfrak{g})_0\)-submodule, where
 160   \(\mathcal{U}(\mathfrak{g})_0\) is the centralizer of \(\mathfrak{h}\) in
 161   \(\mathcal{U}(\mathfrak{g})\) -- which coincides with the weight space of \(0
 162   \in \mathfrak{h}^*\) in the adjoint \(\mathfrak{g}\)-module
 163   \(\mathcal{U}(\mathfrak{g})\), as seen in
 164   Example~\ref{ex:adjoint-action-in-universal-enveloping-is-weight}.
 165 \end{note}
 166 
 167 A particularly well behaved class of examples are the so called
 168 \emph{bounded} modules.
 169 
 170 \begin{definition}\index{\(\mathfrak{g}\)-module!bounded modules}\index{\(\mathfrak{g}\)-module!(essential) support}
 171   A weight \(\mathfrak{g}\)-module \(M\) is called \emph{bounded} if \(\dim
 172   M_\lambda\) is bounded. The lowest upper bound \(\deg M\) for \(\dim
 173   M_\lambda\) is called \emph{the degree of \(M\)}. The \emph{essential
 174   support} of \(M\) is the set \(\operatorname{supp}_{\operatorname{ess}} M =
 175   \{ \lambda \in \mathfrak{h}^* : \dim M_\lambda = \deg M \}\).
 176 \end{definition}
 177 
 178 \begin{example}\label{ex:supp-ess-of-tensor-is-product}
 179   Let \(\mathfrak{g}_1\) and \(\mathfrak{g}_2\) be Lie algebras with Cartan
 180   subalgebras \(\mathfrak{h}_i \subset \mathfrak{g}_i\) and take \(\mathfrak{g}
 181   = \mathfrak{g}_1 \oplus \mathfrak{g}_2\). Given bounded
 182   \(\mathfrak{g}_i\)-modules \(M_i\), it follows from
 183   Example~\ref{ex:tensor-prod-of-weight-is-weight} that \(M_1 \otimes M_2\) is
 184   a bounded \(\mathfrak{g}\)-module with \(\deg M_1 \otimes M_2 = \deg M_1
 185   \cdot \deg M_2\) and
 186   \[
 187     \operatorname{supp}_{\operatorname{ess}} (M_1 \otimes M_2)
 188     = \operatorname{supp}_{\operatorname{ess}} M_1 \oplus
 189       \operatorname{supp}_{\operatorname{ess}} M_2
 190     = \{
 191         \lambda_1 + \lambda_2 : \lambda_i \in
 192         \operatorname{supp}_{\operatorname{ess}} M_i \subset \mathfrak{h}_i^*
 193       \}
 194   \]
 195 \end{example}
 196 
 197 \begin{example}\label{ex:laurent-polynomial-mod}
 198   There is a natural action of \(\mathfrak{sl}_2(K)\) on the space \(K[x,
 199   x^{-1}]\) of Laurent polynomials, given by the formulas in
 200   (\ref{eq:laurent-polynomials-cusp-mod}). One can quickly verify \(K[x,
 201   x^{-1}]_{2 k} = K x^k\) and \(K[x, x^{-1}]_\lambda = 0\) for any \(\lambda
 202   \notin 2 \mathbb{Z}\), so that \(K[x, x^{-1}] = \bigoplus_{k \in \mathbb{Z}}
 203   K x^k\) is a degree \(1\) bounded weight \(\mathfrak{sl}_2(K)\)-module. It
 204   follows from the remark at the end of Example~\ref{ex:submod-is-weight-mod}
 205   that any nonzero submodule \(N \subset K[x, x^{-1}]\) must contain a
 206   monomial \(x^k\). But since the operators \(-\frac{\mathrm{d}}{\mathrm{d}x} +
 207   \frac{x^{-1}}{2}, x^2 \frac{\mathrm{d}}{\mathrm{d}x} + \frac{x}{2} : K[x,
 208   x^{-1}] \to K[x, x^{-1}]\) are both injective, this implies all other
 209   monomials can be found in \(N\) by successively applying \(f\) and \(e\).
 210   Hence \(N = K[x, x^{-1}]\) and \(K[x, x^{-1}]\) is a simple module.
 211   \begin{align}\label{eq:laurent-polynomials-cusp-mod}
 212     e \cdot p
 213     & = \left( x^2 \frac{\mathrm{d}}{\mathrm{d}x} + \frac{x}{2} \right) p &
 214     f \cdot p
 215     & = \left(- \frac{\mathrm{d}}{\mathrm{d}x} + \frac{x^{-1}}{2} \right) p &
 216     h \cdot p
 217     & = 2 x \frac{\mathrm{d}}{\mathrm{d}x} p
 218   \end{align}
 219 \end{example}
 220 
 221 Notice that the support of \(K[x, x^{-1}]\) is the trivial \(2
 222 \mathbb{Z}\)-coset \(0 + 2 \mathbb{Z}\). This is representative of the general
 223 behavior in the following sense: if \(M\) is a simple weight
 224 \(\mathfrak{g}\)-module, since \(M[\lambda] = \bigoplus_{\alpha \in Q}
 225 M_{\lambda + \alpha}\) is stable under the action of \(\mathfrak{g}\) for all
 226 \(\lambda \in \mathfrak{h}^*\), \(\bigoplus_{\alpha \in Q} M_{\lambda +
 227 \alpha}\) is either \(0\) or all of \(M\). In other words, the support of a
 228 simple weight module is always contained in a single \(Q\)-coset.
 229 
 230 However, the behavior of \(K[x, x^{-1}]\) deviates from that of an arbitrary
 231 bounded \(\mathfrak{g}\)-module in the sense its essential support is
 232 precisely the entire \(Q\)-coset it inhabits -- i.e.
 233 \(\operatorname{supp}_{\operatorname{ess}} K[x, x^{-1}] = 2 \mathbb{Z}\). This
 234 isn't always the case. Nevertheless, in general we find\dots
 235 
 236 \begin{proposition}\label{thm:ess-supp-is-zariski-dense}
 237   Let \(\mathfrak{g}\) be a finite-dimensional semisimple Lie algebra and \(M\)
 238   be a simple infinite-dimensional bounded \(\mathfrak{g}\)-module. The
 239   essential support \(\operatorname{supp}_{\operatorname{ess}} M\) is
 240   Zariski-dense\footnote{Any choice of basis for $\mathfrak{h}^*$ induces a
 241   $K$-linear isomorphism $\mathfrak{h}^* \isoto K^n$. In particular, a choice
 242   of basis induces a unique topology in $\mathfrak{h}^*$ such that the map
 243   $\mathfrak{h}^* \to K^n$ is a homeomorphism onto $K^n$ with the Zariski
 244   topology. Any two basis induce the same topology in $\mathfrak{h}^*$, which
 245   we call \emph{the Zariski topology of $\mathfrak{h}^*$}.} in
 246   \(\mathfrak{h}^*\).
 247 \end{proposition}
 248 
 249 This proof was deemed too technical to be included in here, but see Proposition
 250 3.5 of \cite{mathieu} for the case where \(\mathfrak{g} = \mathfrak{s}\) is a
 251 simple Lie algebra. The general case then follows from
 252 Example~\ref{thm:simple-weight-mod-is-tensor-prod},
 253 Example~\ref{ex:supp-ess-of-tensor-is-product} and the asserting that the
 254 product of Zariski-dense subsets in \(K^n\) and \(K^m\) is Zariski-dense in
 255 \(K^{n + m} = K^n \times K^m\).
 256 
 257 We now begin a systematic investigation of the problem of classifying the
 258 infinite-dimensional simple weight modules of a given Lie algebra
 259 \(\mathfrak{g}\). As in the previous chapter, let \(\mathfrak{g}\) be a
 260 finite-dimensional semisimple Lie algebra. As a first approximation of a
 261 solution to our problem, we consider the Verma modules \(M(\lambda)\) for
 262 \(\lambda \in \mathfrak{h}^*\) which is not dominant integral. After all, the
 263 simple quotients of Verma modules form a remarkably large class of
 264 infinite-dimensional simple weight modules -- at least as large as
 265 \(\mathfrak{h}^* \setminus P^+\)! More generally, the induction functor
 266 \(\operatorname{Ind}_{\mathfrak{b}}^{\mathfrak{g}} :
 267 \mathfrak{b}\text{-}\mathbf{Mod} \to \mathfrak{g}\text{-}\mathbf{Mod}\) has
 268 proven itself a powerful tool for constructing modules.
 269 
 270 We claim this is not an unmotivated guess. Specifically, there are very good
 271 reasons behind the choice to consider induction over the Borel subalgebra
 272 \(\mathfrak{b} \subset \mathfrak{g}\). First, the fact that \(\mathfrak{h}
 273 \subset \mathfrak{g}\) affords us great control over the weight spaces of
 274 \(\operatorname{Ind}_{\mathfrak{b}}^{\mathfrak{g}} M\): by assigning a
 275 prescribed action of \(\mathfrak{h}\) to \(M\) we can ensure that
 276 \(\operatorname{Ind}_{\mathfrak{b}}^{\mathfrak{g}} M = \bigoplus_\lambda
 277 (\operatorname{Ind}_{\mathfrak{b}}^{\mathfrak{g}} M)_\lambda\). In addition, we
 278 have seen in the proof of Proposition~\ref{thm:high-weight-mod-is-weight-mod}
 279 that by requiring that the positive part of \(\mathfrak{b}\) acts on \(M\) by
 280 zero we can ensure that \(\dim
 281 (\operatorname{Ind}_{\mathfrak{b}}^{\mathfrak{g}} M)_\lambda < \infty\). All in
 282 all, the nature of \(\mathfrak{b}\) affords us just enough control to guarantee
 283 that \(\operatorname{Ind}_{\mathfrak{b}}^{\mathfrak{g}} M\) is a weight module
 284 for sufficiently well behaved \(M\).
 285 
 286 Unfortunately for us, this is still too little control: there are simple weight
 287 modules which are not of the form \(L(\lambda)\). More generally, we may
 288 consider induction over some parabolic subalgebra \(\mathfrak{p} \subset
 289 \mathfrak{g}\) -- i.e. some subalgebra such that \(\mathfrak{p} \supset
 290 \mathfrak{b}\). This leads us to the following definition.
 291 
 292 \begin{definition}\index{\(\mathfrak{g}\)-module!(generalized) Verma modules}
 293   Let \(\mathfrak{p} \subset \mathfrak{g}\) be a parabolic subalgebra and \(M\)
 294   be a simple \(\mfrac{\mathfrak{p}}{\mathfrak{nil}(\mathfrak{p})}\)-module. We
 295   can view \(M\) as a \(\mathfrak{p}\)-module where
 296   \(\mathfrak{nil}(\mathfrak{p})\) acts by zero by setting \(X \cdot m = (X +
 297   \mathfrak{nil}(\mathfrak{p})) \cdot m\) for all \(m \in M\) and \(X \in
 298   \mathfrak{p}\) -- which is the same as the \(\mathfrak{p}\)-module given by
 299   composing the action map \(\mfrac{\mathfrak{p}}{\mathfrak{nil}(\mathfrak{p})}
 300   \to \mathfrak{gl}(M)\) with the projection \(\mathfrak{p} \to
 301   \mfrac{\mathfrak{p}}{\mathfrak{nil}(\mathfrak{p})}\). The module
 302   \(M_{\mathfrak{p}}(M) = \operatorname{Ind}_{\mathfrak{p}}^{\mathfrak{g}} M\)
 303   is called \emph{generalized Verma module associated with \(M\)}.
 304 \end{definition}
 305 
 306 \begin{example}
 307   It is not hard to see that
 308   \(\mfrac{\mathfrak{b}}{\mathfrak{nil}(\mathfrak{b})} = \mathfrak{h}\). If we
 309   take \(\lambda \in \mathfrak{h}^*\) and let \(K m^+\) be the
 310   \(1\)-dimensional \(\mathfrak{h}\)-module where \(\mathfrak{h}\) acts by
 311   \(\lambda\) then \(M(\lambda) = M_{\mathfrak{b}}(K m^+)\).
 312 \end{example}
 313 
 314 As promised, \(M_{\mathfrak{p}}(M)\) is generally well behaved for well behaved
 315 \(M\). In particular, if \(M\) is highest weight
 316 \(\mfrac{\mathfrak{p}}{\mathfrak{nil}(\mathfrak{p})}\)-module then
 317 \(M_{\mathfrak{p}}(M)\) is also a highest weight \(\mathfrak{g}\)-module, and
 318 if \(M\) is a weight
 319 \(\mfrac{\mathfrak{p}}{\mathfrak{nil}(\mathfrak{p})}\)-module then
 320 \(M_{\mathfrak{p}}(M)\) is a weight module with \(M_{\mathfrak{p}}(M)_\lambda =
 321 \sum_{\alpha + \mu = \lambda} \mathcal{U}(\mathfrak{g})_\alpha
 322 \otimes_{\mathcal{U}(\mathfrak{p})} M_\mu\), -- see Lemma 1.1 of \cite{mathieu}
 323 for a full proof. However, \(M_{\mathfrak{p}}(M)\) is not simple in general.
 324 Indeed, regular Verma modules not necessarily simple. This issue may be dealt
 325 with by passing to the simple quotients of \(M_{\mathfrak{p}}(M)\).
 326 
 327 Let \(M\) be a simple weight
 328 \(\mfrac{\mathfrak{p}}{\mathfrak{nil}(\mathfrak{p})}\)-module. As it turns out,
 329 the situation encountered in Proposition~\ref{thm:max-verma-submod-is-weight}
 330 is also verified in the general setting. Namely, since \(M_{\mathfrak{p}}(M)\)
 331 is generated by \(K \otimes_{\mathcal{U}(\mathfrak{p})} M = \bigoplus_{\lambda
 332 \in Q + \operatorname{supp} M} M_{\mathfrak{p}}(M)_\lambda\), it follows that
 333 any proper submodule of \(M_{\mathfrak{p}}(M)\) is contained in
 334 \(\bigoplus_{\lambda \notin Q + \operatorname{supp} M}
 335 M_{\mathfrak{p}}(M)_\lambda\). The sum \(N_{\mathfrak{p}}(M)\) of all such
 336 submodules is thus the unique maximal submodule of \(M_{\mathfrak{p}}(M)\) and
 337 \(L_{\mathfrak{p}}(M) = \mfrac{M_{\mathfrak{p}}(M)}{N_{\mathfrak{p}}(M)}\) is
 338 its unique simple quotient -- again, we refer the reader to \cite{mathieu} for
 339 a complete proof. This leads us to the following definition.
 340 
 341 \begin{definition}\index{\(\mathfrak{g}\)-module!parabolic induced modules}\index{\(\mathfrak{g}\)-module!cuspidal modules}
 342   A simple weight \(\mathfrak{g}\)-module is called \emph{parabolic induced} if
 343   it is isomorphic to \(L_{\mathfrak{p}}(M)\) for some proper parabolic
 344   subalgebra \(\mathfrak{p} \subsetneq \mathfrak{g}\) and some simple weight
 345   \(\mfrac{\mathfrak{p}}{\mathfrak{nil}(\mathfrak{p})}\)-module \(M\). A
 346   \emph{cuspidal \(\mathfrak{g}\)-module} is a simple weight
 347   \(\mathfrak{g}\)-module which is \emph{not} parabolic induced.
 348 \end{definition}
 349 
 350 The first breakthrough regarding our classification problem was given by
 351 Fernando in his now infamous paper \citetitle{fernando} \cite{fernando}, where
 352 he proved that every simple weight \(\mathfrak{g}\)-module is parabolic
 353 induced by a cuspidal module.
 354 
 355 \begin{theorem}[Fernando]
 356   Any simple weight \(\mathfrak{g}\)-module is isomorphic to
 357   \(L_{\mathfrak{p}}(M)\) for some parabolic subalgebra \(\mathfrak{p} \subset
 358   \mathfrak{g}\) and some cuspidal
 359   \(\mfrac{\mathfrak{p}}{\mathfrak{nil}(\mathfrak{p})}\)-module \(M\).
 360 \end{theorem}
 361 
 362 We should point out that the relationship between simple weight
 363 \(\mathfrak{g}\)-modules and pairs \((\mathfrak{p}, M)\) is not one-to-one.
 364 Nevertheless, this relationship is well understood. Namely, Fernando himself
 365 established\dots
 366 
 367 \begin{proposition}[Fernando]
 368   Given a parabolic subalgebra \(\mathfrak{p} \subset \mathfrak{g}\), there
 369   exists a basis \(\Sigma\) for \(\Delta\) such that \(\Sigma \subset
 370   \Delta_{\mathfrak{p}} \subset \Delta\), where \(\Delta_{\mathfrak{p}}\)
 371   denotes the set of roots of \(\mathfrak{p}\). Furthermore, if \(\mathfrak{p}'
 372   \subset \mathfrak{g}\) is another parabolic subalgebra, \(M\) is a cuspidal
 373   \(\mfrac{\mathfrak{p}}{\mathfrak{nil}(\mathfrak{p})}\)-module and \(N\) is a
 374   cuspidal \(\mfrac{\mathfrak{p}'}{\mathfrak{nil}(\mathfrak{p}')}\)-module then
 375   \(L_{\mathfrak{p}}(M) \cong L_{\mathfrak{p}'}(N)\) if, and only if
 376   \(\mathfrak{p}' = \twisted{\mathfrak{p}}{\sigma}\) and \(M \cong
 377   \twisted{N}{\sigma}\) as \(\mathfrak{p}\)-modules for some\footnote{Here
 378   $\twisted{\mathfrak{p}}{\sigma}$ denotes the image of $\mathfrak{p}$ under
 379   the automorphism of $\sigma : \mathfrak{g} \to \mathfrak{g}$ given by the
 380   canonical action of $W$ on $\mathfrak{g}$ and $\twisted{N}{\sigma}$ is the
 381   $\mathfrak{p}$-module given by composing the map $\mathfrak{p}' \to
 382   \mathfrak{gl}(N)$ with the restriction $\sigma\!\restriction_{\mathfrak{p}} :
 383   \mathfrak{p} \to \mathfrak{p}'$.} \(\sigma \in W_M\), where
 384   \[
 385     W_M
 386     = \langle
 387       \sigma_\beta : \beta \in \Sigma, H_\beta + \mathfrak{nil}(\mathfrak{p})
 388       \ \text{is central in}\ \mfrac{\mathfrak{p}}{\mathfrak{nil}(\mathfrak{p})}
 389       \ \text{and}\ H_\beta\ \text{acts on \(M\) as a positive integer}
 390       \rangle
 391     \subset W
 392   \]
 393 \end{proposition}
 394 
 395 \begin{note}
 396   The definition of the subgroup \(W_M \subset W\) is independent of the choice
 397   of basis \(\Sigma\).
 398 \end{note}
 399 
 400 As a first consequence of Fernando's Theorem, we provide two alternative
 401 characterizations of cuspidal modules.
 402 
 403 \begin{corollary}[Fernando]\label{thm:cuspidal-mod-equivs}
 404   Let \(M\) be a simple weight \(\mathfrak{g}\)-module. The following
 405   conditions are equivalent.
 406   \begin{enumerate}
 407     \item \(M\) is cuspidal.
 408     \item \(F_\alpha\) acts injectively on \(M\) for all
 409       \(\alpha \in \Delta\).
 410     \item The support of \(M\) is precisely one \(Q\)-coset.
 411   \end{enumerate}
 412 \end{corollary}
 413 
 414 \begin{example}
 415   As noted in Example~\ref{ex:laurent-polynomial-mod}, the element \(f \in
 416   \mathfrak{sl}_2(K)\) acts injectively on the space of Laurent polynomials.
 417   Hence \(K[x, x^{-1}]\) is a cuspidal \(\mathfrak{sl}_2(K)\)-module.
 418 \end{example}
 419 
 420 Having reduced our classification problem to that of classifying cuspidal
 421 modules, we are now faced the daunting task of actually classifying them.
 422 Historically, this was first achieved by Olivier Mathieu in the early 2000's in
 423 his paper \citetitle{mathieu} \cite{mathieu}. To do so, Mathieu introduced new
 424 tools which have since proved themselves remarkably useful throughout the
 425 field, known as\dots
 426 
 427 \section{Coherent Families}
 428 
 429 We begin our analysis with a simple question: how to do we go about
 430 constructing cuspidal modules? Specifically, given a cuspidal
 431 \(\mathfrak{g}\)-module, how can we use it to produce new cuspidal modules? To
 432 answer this question, we look back at the single example of a cuspidal module
 433 we have encountered so far: the \(\mathfrak{sl}_2(K)\)-module \(K[x, x^{-1}]\)
 434 of Laurent polynomials -- i.e. Example~\ref{ex:laurent-polynomial-mod}.
 435 
 436 Our first observation is that \(\mathfrak{sl}_2(K)\) acts on \(K[x, x^{-1}]\)
 437 via differential operators. In other words, the action map
 438 \(\mathcal{U}(\mathfrak{sl}_2(K)) \to \operatorname{End}(K[x, x^{-1}])\)
 439 factors through the inclusion of the algebra \(\operatorname{Diff}(K[x,
 440 x^{-1}]) = K\left[x, x^{-1}, \frac{\mathrm{d}}{\mathrm{d}x}\right]\) of
 441 differential operators in \(K[x, x^{-1}]\).
 442 \begin{center}
 443   \begin{tikzcd}
 444     \mathcal{U}(\mathfrak{sl}_2(K))   \rar &
 445     \operatorname{Diff}(K[x, x^{-1}]) \rar &
 446     \operatorname{End}(K[x, x^{-1}])
 447   \end{tikzcd}
 448 \end{center}
 449 
 450 The space \(K[x, x^{-1}]\) can be regarded as a \(\operatorname{Diff}(K[x,
 451 x^{-1}])\)-module in the natural way, and we can produce new
 452 \(\operatorname{Diff}(K[x, x^{-1}])\)-modules by twisting \(K[x, x^{-1}]\) by
 453 automorphisms of \(\operatorname{Diff}(K[x, x^{-1}])\). For example, given
 454 \(\lambda \in K\) we may take the automorphism
 455 \begin{align*}
 456   \varphi_\lambda : \operatorname{Diff}(K[x, x^{-1}]) &
 457   \to \operatorname{Diff}(K[x, x^{-1}]) \\
 458   x & \mapsto x \\
 459   x^{-1} & \mapsto x^{-1} \\
 460   \frac{\mathrm{d}}{\mathrm{d} x} & \mapsto \frac{\mathrm{d}}{\mathrm{d} x} +
 461   \frac{\lambda}{2} x^{-1}
 462 \end{align*}
 463 and consider the twisted module \(\twisted{K[x, x^{-1}]}{\varphi_\lambda} =
 464 K[x, x^{-1}]\), where some operator \(P \in \operatorname{Diff}(K[x, x^{-1}])\)
 465 acts as \(\varphi_\lambda(P)\).
 466 
 467 By composing the action map \(\operatorname{Diff}(K[x, x^{-1}]) \to
 468 \operatorname{End}(\twisted{K[x, x^{-1}]}{\varphi_\lambda})\) with the
 469 homomorphism of algebras \(\mathcal{U}(\mathfrak{sl}_2(K)) \to
 470 \operatorname{Diff}(K[x, x^{-1}])\) we can give \(\twisted{K[x,
 471 x^{-1}]}{\varphi_\lambda}\) the structure of an \(\mathfrak{sl}_2(K)\)-module.
 472 Diagrammatically, we have
 473 \begin{center}
 474   \begin{tikzcd}
 475     \mathcal{U}(\mathfrak{sl}_2(K))   \rar                  &
 476     \operatorname{Diff}(K[x, x^{-1}]) \rar{\varphi_\lambda} &
 477     \operatorname{Diff}(K[x, x^{-1}]) \rar                  &
 478     \operatorname{End}(K[x, x^{-1}])
 479   \end{tikzcd},
 480 \end{center}
 481 where the maps \(\mathcal{U}(\mathfrak{sl}_2(K)) \to \operatorname{Diff}(K[x,
 482 x^{-1}])\) and \(\operatorname{Diff}(K[x, x^{1}]) \to \operatorname{End}(K[x,
 483 x^{-1}])\) are the ones from the previous diagram.
 484 
 485 Explicitly, we find that the action of \(\mathfrak{sl}_2(K)\) on
 486 \(\twisted{K[x, x^{-1}]}{\varphi_\lambda}\) is given by
 487 \begin{align*}
 488   p & \overset{e}{\mapsto}
 489   \left(
 490   x^2 \frac{\mathrm{d}}{\mathrm{d}x} + \frac{1 + \lambda}{2} x
 491   \right) p &
 492   p & \overset{f}{\mapsto}
 493   \left(
 494   - \frac{\mathrm{d}}{\mathrm{d}x} + \frac{1 - \lambda}{2} x^{-1}
 495   \right) p &
 496   p & \overset{h}{\mapsto}
 497   \left( 2 x \frac{\mathrm{d}}{\mathrm{d}x} + \lambda \right) p,
 498 \end{align*}
 499 so we can see \(\twisted{K[x, x^{-1}]}{\varphi_\lambda}_{2 k +
 500 \frac{\lambda}{2}} = K x^k\) for all \(k \in \mathbb{Z}\) and \(\twisted{K[x,
 501 x^{-1}]}{\varphi_\lambda}_\mu = 0\) for all other \(\mu \in \mathfrak{h}^*\).
 502 
 503 Hence \(\twisted{K[x, x^{-1}]}{\varphi_\lambda}\) is a degree \(1\) bounded
 504 \(\mathfrak{sl}_2(K)\)-module with \(\operatorname{supp} \twisted{K[x,
 505 x^{-1}]}{\varphi_\lambda} = \frac{\lambda}{2} + 2 \mathbb{Z}\). One can also
 506 quickly check that if \(\lambda \notin 1 + 2 \mathbb{Z}\) then \(e\) and \(f\)
 507 act injectively in \(\twisted{K[x, x^{-1}]}{\varphi_\lambda}\), so that
 508 \(\twisted{K[x, x^{-1}]}{\varphi_\lambda}\) is simple. In particular, if
 509 \(\lambda, \mu \notin 1 + 2 \mathbb{Z}\) with \(\lambda \notin \mu + 2
 510 \mathbb{Z}\) then \(\twisted{K[x, x^{-1}]}{\varphi_\lambda}\) and
 511 \(\twisted{K[x, x^{-1}]}{\varphi_\mu}\) are non-isomorphic cuspidal
 512 \(\mathfrak{sl}_2(K)\)-modules, since their supports differ. These cuspidal
 513 modules can be ``glued together'' in a \emph{monstrous concoction} by summing
 514 over \(\lambda \in K\), as in
 515 \[
 516   \mathcal{M}
 517   = \bigoplus_{\lambda + 2 \mathbb{Z} \in \mfrac{K}{2 \mathbb{Z}}}
 518     \twisted{K[x, x^{-1}]}{\varphi_\lambda},
 519 \]
 520 
 521 To a distracted spectator, \(\mathcal{M}\) may look like just another,
 522 innocent, \(\mathfrak{sl}_2(K)\)-module. However, the attentive reader may have
 523 already noticed some of the its bizarre features, most noticeable of which is
 524 the fact that \(\mathcal{M}\) is very big. In fact, \(\mathcal{M}\) is as big a
 525 degree \(1\) bounded module gets: \(\operatorname{supp} \mathcal{M}
 526 = \operatorname{supp}_{\operatorname{ess}} \mathcal{M}\) is the entirety of
 527 \(\mathfrak{h}^*\). This may look very alien the reader familiarized with the
 528 finite-dimensional setting, where the configuration of weights is very rigid.
 529 For this reason, \(\mathcal{M}\) deserves to be called ``a monstrous
 530 concoction''.
 531 
 532 On a perhaps less derogatory note, \(\mathcal{M}\) also deserves to be called
 533 \emph{a family}. This is because \(\mathcal{M}\) consists of lots of smaller
 534 cuspidal modules which fit together inside of it in a \emph{coherent} fashion.
 535 Mathieu's ingenious breakthrough was the realization that \(\mathcal{M}\) is a
 536 particular example of a more general pattern, which he named \emph{coherent
 537 families}.
 538 
 539 \begin{definition}\index{coherent family}
 540   A \emph{coherent family \(\mathcal{M}\) of degree \(d\)} is a weight
 541   \(\mathfrak{g}\)-module \(\mathcal{M}\) such that
 542   \begin{enumerate}
 543     \item \(\dim \mathcal{M}_\lambda = d\) for \emph{all} \(\lambda \in
 544       \mathfrak{h}^*\) -- i.e. \(\operatorname{supp}_{\operatorname{ess}}
 545       \mathcal{M} = \mathfrak{h}^*\).
 546     \item For any \(u \in \mathcal{U}(\mathfrak{g})\) in the centralizer
 547       \(\mathcal{U}(\mathfrak{g})_0\) of \(\mathfrak{h}\) in
 548       \(\mathcal{U}(\mathfrak{g})\), the map
 549       \begin{align*}
 550         \mathfrak{h}^* & \to K \\
 551                \lambda & \mapsto
 552                \operatorname{Tr}(u\!\restriction_{\mathcal{M}_\lambda})
 553       \end{align*}
 554       is polynomial in \(\lambda\).
 555   \end{enumerate}
 556 \end{definition}
 557 
 558 \begin{example}\label{ex:sl-laurent-family}
 559   The module \(\mathcal{M} = \bigoplus_{\lambda + 2 \mathbb{Z} \in
 560   \mfrac{K}{2 \mathbb{Z}}} \twisted{K[x, x^{-1}]}{\varphi_\lambda}\) is a
 561   degree \(1\) coherent \(\mathfrak{sl}_2(K)\)-family.
 562 \end{example}
 563 
 564 \begin{example}
 565   Given \(\lambda \in K\), \(\mathcal{M}(\lambda) = \bigoplus_{\mu \in K} K
 566   x^\mu\) with
 567   \begin{align*}
 568     p & \overset{e}{\mapsto}
 569         \left(x^2 \frac{\mathrm{d}}{\mathrm{d}x} + \lambda x\right) p &
 570     p & \overset{f}{\mapsto}
 571         \left(-\frac{\mathrm{d}}{\mathrm{d}x} + \lambda x^{-1}\right) p &
 572     p & \overset{h}{\mapsto} 2 x \frac{\mathrm{d}}{\mathrm{d}x} p,
 573   \end{align*}
 574   is a degree \(1\) coherent \(\mathfrak{sl}_2(K)\)-family -- where \(x^{\pm
 575   1}, \sfrac{\mathrm{d}}{\mathrm{d}x} : \mathcal{M}(\lambda) \to
 576   \mathcal{M}(\lambda)\) are given by \(x^{\pm 1} x^\mu = x^{\mu \pm 1}\) and
 577   \(\sfrac{\mathrm{d}}{\mathrm{d}x} x^\mu = \mu x^{\mu - 1}\). It is easy to
 578   check \(\mathcal{M}\) from Example~\ref{ex:sl-laurent-family} is isomorphic
 579   to \(\mathcal{M}(\sfrac{1}{2})\) and \((\mathcal{M}(\sfrac{1}{2}))[0] \cong
 580   K[x, x^{-1}]\).
 581 \end{example}
 582 
 583 \begin{note}
 584   We would like to stress that coherent families have proven themselves useful
 585   for problems other than the classification of cuspidal
 586   \(\mathfrak{g}\)-modules. For instance, Nilsson's classification of rank 1
 587   \(\mathfrak{h}\)-free \(\mathfrak{sp}_{2 n}(K)\)-modules is based on the
 588   notion of coherent families and the so called \emph{weighting functor}.
 589 \end{note}
 590 
 591 Our hope is that given a cuspidal module \(M\), we can somehow fit \(M\) inside
 592 of a coherent \(\mathfrak{g}\)-family, such as in the case of \(K[x, x^{-1}]\)
 593 and \(\mathcal{M}\) from Example~\ref{ex:sl-laurent-family}. In addition, we
 594 hope that such coherent families are somehow \emph{uniquely determined} by
 595 \(M\). This leads us to the following definition.
 596 
 597 \begin{definition}\index{coherent family!coherent extension}
 598   Given a bounded \(\mathfrak{g}\)-module \(M\) of degree \(d\), a
 599   \emph{coherent extension \(\mathcal{M}\) of \(M\)} is a coherent family
 600   \(\mathcal{M}\) of degree \(d\) that contains \(M\) as a subquotient.
 601 \end{definition}
 602 
 603 Our goal is now showing that every simple bounded module has a coherent
 604 extension. The idea then is to classify coherent families, and classify which
 605 submodules of a given coherent family are actually cuspidal modules. If every
 606 simple bounded \(\mathfrak{g}\)-module fits inside a coherent extension, this
 607 would lead to classification of all cuspidal \(\mathfrak{g}\)-modules, which we
 608 now know is the key for the solution of our classification problem. However,
 609 there are some complications to this scheme.
 610 
 611 Leaving aside the question of existence for a second, we should point out that
 612 coherent families turn out to be rather complicated on their own. In fact they
 613 are too complicated to classify in general. Ideally, we would like to find
 614 \emph{nice} coherent extensions -- ones we can actually classify. For instance,
 615 we may search for \emph{irreducible} coherent extensions, which are defined as
 616 follows.
 617 
 618 \begin{definition}\index{coherent family!irreducible coherent family}
 619   A coherent family \(\mathcal{M}\) is called \emph{irreducible} if it contains
 620   no proper coherent subfamilies -- i.e. \(\mathcal{M}\) is a simple object in
 621   the full subcategory of \(\mathfrak{g}\text{-}\mathbf{Mod}\) consisting of
 622   coherent families. Equivalently, we call \(\mathcal{M}\) irreducible if
 623   \(\mathcal{M}_\lambda\) is a simple \(\mathcal{U}(\mathfrak{g})_0\)-module
 624   for some \(\lambda \in \mathfrak{h}^*\).
 625 \end{definition}
 626 
 627 Another natural candidate for the role of ``nice extensions'' are the
 628 semisimple coherent families -- i.e. families which are semisimple as
 629 \(\mathfrak{g}\)-modules. These turn out to be very easy to produce. Namely,
 630 there is a construction, known as \emph{the semisimplification of a coherent
 631 family}, which takes a coherent extension of \(M\) to a semisimple coherent
 632 extension of \(M\).
 633 
 634 % Mathieu's proof of this is somewhat profane, I don't think it's worth
 635 % including it in here
 636 % TODO: Move this somewhere else? This holds in general for weight modules
 637 % whose suppert is contained in a single Q-coset
 638 \begin{lemma}\label{thm:component-coh-family-has-finite-length}
 639   Given a coherent family \(\mathcal{M}\) and \(\lambda \in \mathfrak{h}^*\),
 640   \(\mathcal{M}[\lambda]\) has finite length as a \(\mathfrak{g}\)-module.
 641 \end{lemma}
 642 
 643 \begin{proposition}\index{coherent family!semisimplification}
 644   Let \(\mathcal{M}\) be a coherent family of degree \(d\). There exists a
 645   unique semisimple coherent family \(\mathcal{M}^{\operatorname{ss}}\) of
 646   degree \(d\) such that the composition series of
 647   \(\mathcal{M}^{\operatorname{ss}}[\lambda]\) is the same as that of
 648   \(\mathcal{M}[\lambda]\) for all \(\lambda \in \mathfrak{h}^*\), called
 649   \emph{the semisimplification of \(\mathcal{M}\)}.
 650 
 651   Namely, if \(\lambda \in \mathfrak{h}^*\) and \(0 = \mathcal{M}_{\lambda 0}
 652   \subset \mathcal{M}_{\lambda 1} \subset \cdots \subset \mathcal{M}_{\lambda
 653   r_\lambda} = \mathcal{M}[\lambda]\) is a composition series\footnote{Notice
 654   that $\mathcal{M}[\lambda] = \mathcal{M}[\mu]$ for any $\mu \in \lambda + Q$.
 655   Hence the sum $\bigoplus_{\lambda + Q \in \mfrac{\mathfrak{h}^*}{Q}}
 656   \bigoplus_i \mfrac{\mathcal{M}_{\lambda i + 1}}{\mathcal{M}_{\lambda i}}$ is
 657   independent of the choice of representative for $\lambda + Q$ -- provided we
 658   choose $\mathcal{M}_{\mu i} = \mathcal{M}_{\lambda i}$ for all $\mu \in
 659   \lambda + Q$ and $i$.},
 660   \[
 661     \mathcal{M}^{\operatorname{ss}}
 662     \cong \bigoplus_{\substack{\lambda + Q \in \mfrac{\mathfrak{h}^*}{Q} \\ i}}
 663           \mfrac{\mathcal{M}_{\lambda i + 1}}{\mathcal{M}_{\lambda i}}
 664   \]
 665 \end{proposition}
 666 
 667 \begin{proof}
 668   The uniqueness of \(\mathcal{M}^{\operatorname{ss}}\) should be clear:
 669   since \(\mathcal{M}^{\operatorname{ss}}\) is semisimple, so is
 670   \(\mathcal{M}^{\operatorname{ss}}[\lambda]\). Hence by the Jordan-Hölder
 671   Theorem
 672   \[
 673     \mathcal{M}^{\operatorname{ss}}[\lambda]
 674     \cong
 675     \bigoplus_i \mfrac{\mathcal{M}_{\lambda i + 1}}{\mathcal{M}_{\lambda i}}
 676   \]
 677 
 678   As for the existence of the semisimplification, it suffices to show
 679   \[
 680     \mathcal{M}^{\operatorname{ss}}
 681     = \bigoplus_{\substack{\lambda + Q \in \mfrac{\mathfrak{h}^*}{Q} \\ i}}
 682     \mfrac{\mathcal{M}_{\lambda i + 1}}{\mathcal{M}_{\lambda i}}
 683   \]
 684   is indeed a semisimple coherent family of degree \(d\).
 685 
 686   We know from Examples~\ref{ex:submod-is-weight-mod} and
 687   \ref{ex:quotient-is-weight-mod} that each quotient
 688   \(\mfrac{\mathcal{M}_{\lambda i + 1}}{\mathcal{M}_{\lambda i}}\) is a weight
 689   module. Hence \(\mathcal{M}^{\operatorname{ss}}\) is a weight module.
 690   Furthermore, given \(\mu \in \mathfrak{h}^*\)
 691   \[
 692     \mathcal{M}_\mu^{\operatorname{ss}}
 693     = \bigoplus_{\substack{\lambda + Q \in \mfrac{\mathfrak{h}^*}{Q} \\ i}}
 694       \left(
 695       \mfrac{\mathcal{M}_{\lambda i + 1}}{\mathcal{M}_{\lambda i}}
 696       \right)_\mu
 697     = \bigoplus_i
 698       \left(
 699       \mfrac{\mathcal{M}_{\mu i + 1}}{\mathcal{M}_{\mu i}}
 700       \right)_\mu
 701     \cong \bigoplus_i
 702       \mfrac{(\mathcal{M}_{\mu i + 1})_\mu}
 703             {(\mathcal{M}_{\mu i})_\mu}
 704   \]
 705 
 706   In particular,
 707   \[
 708     \dim \mathcal{M}_\mu^{\operatorname{ss}}
 709     = \sum_i
 710       \dim (\mathcal{M}_{\mu i + 1})_\mu - \dim (\mathcal{M}_{\mu i})_\mu
 711     = \dim \mathcal{M}[\mu]_\mu
 712     = \dim \mathcal{M}_\mu
 713     = d
 714   \]
 715 
 716   Likewise, given \(u \in \mathcal{U}(\mathfrak{g})_0\) the value
 717   \[
 718     \operatorname{Tr}(u\!\restriction_{\mathcal{M}_\mu^{\operatorname{ss}}})
 719     = \sum_i
 720       \operatorname{Tr}(u\!\restriction_{(\mathcal{M}_{\mu i + 1})_\mu})
 721     - \operatorname{Tr}(u\!\restriction_{(\mathcal{M}_{\mu i})_\mu})
 722     = \operatorname{Tr}(u\!\restriction_{\mathcal{M}[\mu]_\mu})
 723     = \operatorname{Tr}(u\!\restriction_{\mathcal{M}_\mu})
 724   \]
 725   is polynomial in \(\mu \in \mathfrak{h}^*\).
 726 \end{proof}
 727 
 728 \begin{note}
 729   Although we have provided an explicit construction of
 730   \(\mathcal{M}^{\operatorname{ss}}\) in terms of \(\mathcal{M}\), we should
 731   point out this construction is not functorial. First, given a
 732   \(\mathfrak{g}\)-homomorphism \(f : \mathcal{M} \to \mathcal{N}\) between
 733   coherent families, it is unclear what \(f^{\operatorname{ss}} :
 734   \mathcal{M}^{\operatorname{ss}} \to \mathcal{N}^{\operatorname{ss}}\) is
 735   supposed to be. Secondly, and this is more relevant, our construction depends
 736   on the choice of composition series \(0 = \mathcal{M}_{\lambda 0} \subset
 737   \cdots \subset \mathcal{M}_{\lambda r_\lambda} = \mathcal{M}[\lambda]\).
 738   While different choices of composition series yield isomorphic results, there
 739   is no canonical isomorphism. In addition, there is no canonical choice of
 740   composition series.
 741 \end{note}
 742 
 743 The proof of Lemma~\ref{thm:component-coh-family-has-finite-length} is
 744 extremely technical and will not be included in here. It suffices to note that,
 745 as in Proposition~\ref{thm:ess-supp-is-zariski-dense}, the general case follows
 746 from the case where \(\mathfrak{g}\) is simple, which may be found in
 747 \cite{mathieu} -- see Lemma 3.3. As promised, if \(\mathcal{M}\) is a coherent
 748 extension of \(M\) then so is \(\mathcal{M}^{\operatorname{ss}}\).
 749 
 750 \begin{proposition}
 751   Let \(M\) be a simple bounded \(\mathfrak{g}\)-module and \(\mathcal{M}\)
 752   be a coherent extension of \(M\). Then \(\mathcal{M}^{\operatorname{ss}}\) is
 753   a coherent extension of \(M\), and \(M\) is in fact a submodule of
 754   \(\mathcal{M}^{\operatorname{ss}}\).
 755 \end{proposition}
 756 
 757 \begin{proof}
 758   Since \(M\) is simple, its support is contained in a single \(Q\)-coset.
 759   This implies that \(M\) is a subquotient of \(\mathcal{M}[\lambda]\) for any
 760   \(\lambda \in \operatorname{supp} M\). If we fix some composition series \(0
 761   = \mathcal{M}_0 \subset \mathcal{M}_1 \subset \cdots \subset \mathcal{M}_r =
 762   \mathcal{M}[\lambda]\) of \(\mathcal{M}[\lambda]\) with \(M \cong
 763   \mfrac{\mathcal{M}_{i + 1}}{\mathcal{M}_i}\), there is a natural inclusion
 764   \[
 765     M
 766     \isoto \mfrac{\mathcal{M}_{i + 1}}{\mathcal{M}_i}
 767     \to \bigoplus_j \mfrac{\mathcal{M}_{j + 1}}{\mathcal{M}_j}
 768     \cong \mathcal{M}^{\operatorname{ss}}[\lambda]
 769   \]
 770 \end{proof}
 771 
 772 Given the uniqueness of the semisimplification, the semisimplification of any
 773 semisimple coherent extension \(\mathcal{M}\) is \(\mathcal{M}\)
 774 itself and therefore\dots
 775 
 776 \begin{corollary}\label{thm:bounded-is-submod-of-extension}
 777   Let \(M\) be a simple bounded \(\mathfrak{g}\)-module and \(\mathcal{M}\)
 778   be a semisimple coherent extension of \(M\). Then \(M\) is
 779   contained in \(\mathcal{M}\).
 780 \end{corollary}
 781 
 782 These last results provide a partial answer to the question of existence of
 783 well behaved coherent extensions. As for the uniqueness \(\mathcal{M}\) in
 784 Corollary~\ref{thm:bounded-is-submod-of-extension}, it suffices to show that
 785 the multiplicities of the simple weight \(\mathfrak{g}\)-modules in
 786 \(\mathcal{M}\) are uniquely determined by \(M\). These multiplicities may be
 787 computed via the following lemma.
 788 
 789 \begin{lemma}\label{thm:centralizer-multiplicity}
 790   Let \(M\) be a semisimple weight \(\mathfrak{g}\)-module. Then \(M_\lambda\)
 791   is a semisimple \(\mathcal{U}(\mathfrak{g})_0\)-module for any \(\lambda \in
 792   \operatorname{supp} M\). Moreover, if \(L\) is a simple weight
 793   \(\mathfrak{g}\)-module such that \(\lambda \in \operatorname{supp} L\) then
 794   \(L_\lambda\) is a simple \(\mathcal{U}(\mathfrak{g})_0\)-module and the
 795   multiplicity \(L\) in \(M\) coincides with the multiplicity of \(L_\lambda\)
 796   in \(M_\lambda\) as a \(\mathcal{U}(\mathfrak{g})_0\)-module.
 797 \end{lemma}
 798 
 799 \begin{proof}
 800   We begin by showing that \(L_\lambda\) is simple. Let \(N \subset L_\lambda\)
 801   be a nontrivial \(\mathcal{U}(\mathfrak{g})_0\)-submodule. We want to
 802   establish that \(N = L_\lambda\).
 803 
 804   If \(\mathcal{U}(\mathfrak{g})_\alpha\) denotes the root space of \(\alpha\)
 805   in \(\mathcal{U}(\mathfrak{g})\) under the adjoint action of \(\mathfrak{g}\)
 806   as in Example~\ref{ex:adjoint-action-in-universal-enveloping-is-weight},
 807   \(\alpha \in Q\), a simple calculation shows
 808   \(\mathcal{U}(\mathfrak{g})_\alpha \cdot N \subset L_{\lambda + \alpha}\).
 809   Since \(L\) is simple and \(N\) is nonzero, it follows from
 810   Example~\ref{ex:adjoint-action-in-universal-enveloping-is-weight} that
 811   \[
 812     L
 813     = \mathcal{U}(\mathfrak{g}) \cdot N
 814     = \bigoplus_{\alpha \in Q} \mathcal{U}(\mathfrak{g})_\alpha \cdot N
 815   \]
 816   and thus \(L_{\lambda + \alpha} = \mathcal{U}(\mathfrak{g})_\alpha \cdot N\).
 817   In particular, \(L_\lambda = \mathcal{U}(\mathfrak{g})_0 \cdot N \subset N\)
 818   and \(N = L_\lambda\).
 819 
 820   Now given a semisimple weight \(\mathfrak{g}\)-module \(M = \bigoplus_i M_i\)
 821   with \(M_i\) simple, it is clear \(M_\lambda = \bigoplus_i (M_i)_\lambda\).
 822   Each \((M_i)_\lambda\) is either \(0\) or a simple
 823   \(\mathcal{U}(\mathfrak{g})_0\)-module, so that \(M_\lambda\) is a semisimple
 824   \(\mathcal{U}(\mathfrak{g})_0\)-module. In addition, to see that the
 825   multiplicity of \(L\) in \(M\) coincides with the multiplicity of
 826   \(L_\lambda\) in \(M_\lambda\) it suffices to show that if \((M_i)_\lambda
 827   \cong (M_j)_\lambda\) are both nonzero then \(M_i \cong M_j\).
 828 
 829   If \(I(M_i) = \mathcal{U}(\mathfrak{g}) \otimes_{\mathcal{U}(\mathfrak{g})_0}
 830   (M_i)_\lambda\), the inclusion of \(\mathcal{U}(\mathfrak{g})_0\)-modules
 831   \((M_i)_\lambda \to M_i\) induces a \(\mathfrak{g}\)-homomorphism
 832   \begin{align*}
 833             I(M_i) & \to     M_i       \\
 834     u \otimes m & \mapsto u \cdot m
 835   \end{align*}
 836 
 837   Since \(M_i\) is simple and \(\lambda \in \operatorname{supp} M_i\), \(M_i =
 838   \mathcal{U}(\mathfrak{g}) \cdot (M_i)_\lambda\). The homomorphism \(I(M_i)
 839   \to M_i\) is thus surjective. Similarly, if \(I(M_j) =
 840   \mathcal{U}(\mathfrak{g}) \otimes_{\mathcal{U}(\mathfrak{g})_0}
 841   (M_j)_\lambda\) then there is a natural surjective
 842   \(\mathfrak{g}\)-homomorphism \(I(M_j) \to M_j\). Now suppose there is an
 843   isomorphism of \(\mathcal{U}(\mathfrak{g})_0\)-modules \(f: (M_i)_\lambda
 844   \isoto (M_j)_\lambda\). Such an isomorphism induces an isomorphism of
 845   \(\mathfrak{g}\)-modules
 846   \begin{align*}
 847     \tilde f : I(M_i) & \isoto  I(M_j)            \\
 848        u \otimes m & \mapsto u \otimes f(m)
 849   \end{align*}
 850 
 851   By composing \(\tilde f\) with the projection \(I(M_j) \to M_j\) we get a
 852   surjective homomorphism \(I(M_i) \to M_j\). We claim \(\ker (I(M_i) \to M_i)
 853   = \ker (I(M_i) \to M_j)\).  To see this, notice that \(\ker(I(M_i) \to M_i)\)
 854   coincides with the largest submodule \(Z(M_i) \subset I(M_i)\) contained in
 855   \(\bigoplus_{\alpha \ne 0} \mathcal{U}(\mathfrak{g})_\alpha
 856   \otimes_{\mathcal{U}(\mathfrak{g})_0} (M_i)_\lambda\). Indeed, a simple
 857   computation shows \(\ker (I(M_i) \to M_i) \cap (\mathcal{U}(\mathfrak{g})_0
 858   \otimes_{\mathcal{U}(\mathfrak{g})_0} (M_i)_\lambda) = 0\), which implies
 859   \(\ker(I(M_i) \to M_i) \subset Z(M_i)\). Since \(M_i\) is simple, \(\ker
 860   (I(M_i) \to M_i)\) is maximal and thus \(\ker(I(M_i) \to M_i) = Z(M_i)\). By
 861   the same token, \(\ker (I(M_j) \to M_j)\) is the largest submodule of
 862   \(I(M_j)\) contained in \(\bigoplus_{\alpha \ne 0}
 863   \mathcal{U}(\mathfrak{g})_\alpha \otimes_{\mathcal{U}(\mathfrak{g})_0}
 864   (M_j)_\lambda\) and therefore \(\ker(I(M_i) \to M_i) =
 865   \tilde{f}^{-1}(\ker(I(M_j) \to M_j)) = \ker(I(M_i) \to M_j)\).
 866 
 867   Hence there is an isomorphism \(\mfrac{I(M_i)}{\ker(I(M_i) \to M_i)} \isoto
 868   M_j\) satisfying
 869   \begin{center}
 870     \begin{tikzcd}
 871       I(M_i) \rar{\tilde f} \dar & I(M_j) \dar \\
 872       \mfrac{I(M_i)}{\ker(I(M_i) \to M_i)} \rar{\sim} & M_j
 873     \end{tikzcd}
 874   \end{center}
 875  and finally \(M_i \cong \mfrac{I(M_i)}{\ker(I(M_i) \to M_i)} \cong M_j\).
 876 \end{proof}
 877 
 878 A complementary question now is: which submodules of a \emph{nice} coherent
 879 family are cuspidal?
 880 
 881 \begin{proposition}[Mathieu]
 882   Let \(\mathcal{M}\) be an irreducible coherent family of degree \(d\) and
 883   \(\lambda \in \mathfrak{h}^*\). The following conditions are equivalent.
 884   \begin{enumerate}
 885     \item \(\mathcal{M}[\lambda]\) is simple.
 886     \item \(F_\alpha\!\restriction_{\mathcal{M}[\lambda]}\) is injective for
 887       all \(\alpha \in \Delta\).
 888     \item \(\mathcal{M}[\lambda]\) is cuspidal.
 889   \end{enumerate}
 890 \end{proposition}
 891 
 892 \begin{proof}
 893   The fact that \strong{(i)} and \strong{(iii)} are equivalent follows directly
 894   from Corollary~\ref{thm:cuspidal-mod-equivs}. Likewise, it is clear from the
 895   corollary that \strong{(iii)} implies \strong{(ii)}. All it is left is to
 896   show \strong{(ii)} implies \strong{(iii)}. This isn't already clear from
 897   Corollary~\ref{thm:cuspidal-mod-equivs} because, at first glance,
 898   $\mathcal{M}[\lambda]$ may not be simple for some $\lambda$ satisfying
 899   \strong{(ii)}. We will show this is never the case.
 900 
 901   Suppose \(F_\alpha\) acts injectively on the submodule
 902   \(\mathcal{M}[\lambda]\), for all \(\alpha \in \Delta\). Since
 903   \(\mathcal{M}[\lambda]\) has finite length, \(\mathcal{M}[\lambda]\) contains
 904   an infinite-dimensional simple \(\mathfrak{g}\)-submodule \(M\). Moreover,
 905   again by Corollary~\ref{thm:cuspidal-mod-equivs} we conclude \(M\) is a
 906   cuspidal module, and its degree is bounded by \(d\). We want to show
 907   \(\mathcal{M}[\lambda] = M\).
 908 
 909   We claim the set \(U = \{\mu \in \mathfrak{h}^* : \mathcal{M}_\mu \ \text{is
 910   a simple $\mathcal{U}(\mathfrak{g})_0$-module}\}\) is Zariski-open. If we
 911   suppose this is the case for a moment or two, it follows from the fact that
 912   \(M\) is simple and \(\operatorname{supp}_{\operatorname{ess}} M\) is
 913   Zariski-dense that \(U \cap \operatorname{supp}_{\operatorname{ess}} M\) is
 914   non-empty. In other words, there is some \(\mu \in \mathfrak{h}^*\) such that
 915   \(\mathcal{M}_\mu\) is a simple \(\mathcal{U}(\mathfrak{g})_0\)-module and
 916   \(\dim M_\mu = \deg M\).
 917 
 918   In particular, \(M_\mu \ne 0\), so \(M_\mu = \mathcal{M}_\mu\). Now given any
 919   simple \(\mathfrak{g}\)-module \(L\), it follows from
 920   Lemma~\ref{thm:centralizer-multiplicity} that the multiplicity of \(L\)
 921   in \(\mathcal{M}[\lambda]\) is the same as the multiplicity \(L_\mu\) in
 922   \(\mathcal{M}_\mu\) as a \(\mathcal{U}(\mathfrak{g})_0\)-module -- which is,
 923   of course, \(1\) if \(L \cong M\) and \(0\) otherwise. Hence
 924   \(\mathcal{M}[\lambda] = M\) and \(\mathcal{M}[\lambda]\) is cuspidal.
 925 \end{proof}
 926 
 927 To finish the proof, we now show\dots
 928 
 929 \begin{lemma}\label{thm:set-of-simple-u0-mods-is-open}
 930   Let \(\mathcal{M}\) be a coherent family. The set \(U = \{\lambda \in
 931   \mathfrak{h}^* : \mathcal{M}_\lambda \ \text{is a simple
 932   $\mathcal{U}(\mathfrak{g})_0$-module}\}\) is Zariski-open.
 933 \end{lemma}
 934 
 935 \begin{proof}
 936   For each \(\lambda \in \mathfrak{h}^*\) we introduce the bilinear form
 937   \begin{align*}
 938     B_\lambda : \mathcal{U}(\mathfrak{g})_0 \times \mathcal{U}(\mathfrak{g})_0
 939     & \to K \\
 940     (u, v)
 941     & \mapsto \operatorname{Tr}(u v \!\restriction_{\mathcal{M}_\lambda})
 942   \end{align*}
 943   and consider its rank -- i.e. the dimension of the image of the induced
 944   operator
 945   \begin{align*}
 946     \mathcal{U}(\mathfrak{g})_0 & \to     \mathcal{U}(\mathfrak{g})_0^* \\
 947                               u & \mapsto B_\lambda(u, \cdot)
 948   \end{align*}
 949 
 950   Our first observation is that \(\operatorname{rank} B_\lambda \le d^2\). This
 951   follows from the commutativity of
 952   \begin{center}
 953     \begin{tikzcd}
 954       \mathcal{U}(\mathfrak{g})_0               \rar \dar  &
 955       \mathcal{U}(\mathfrak{g})_0^*                        \\
 956       \operatorname{End}(\mathcal{M}_\lambda)   \rar{\sim} &
 957       \operatorname{End}(\mathcal{M}_\lambda)^* \uar
 958     \end{tikzcd},
 959   \end{center}
 960   where the map \(\mathcal{U}(\mathfrak{g})_0 \to
 961   \operatorname{End}(\mathcal{M}_\lambda)\) is given by the action of
 962   \(\mathcal{U}(\mathfrak{g})_0\), the map
 963   \(\operatorname{End}(\mathcal{M}_\lambda)^* \to
 964   \mathcal{U}(\mathfrak{g})_0^*\) is its dual, and the isomorphism
 965   \(\operatorname{End}(\mathcal{M}_\lambda) \isoto
 966   \operatorname{End}(\mathcal{M}_\lambda)^*\) is induced by the trace form
 967   \begin{align*}
 968     \operatorname{End}(\mathcal{M}_\lambda) \times
 969     \operatorname{End}(\mathcal{M}_\lambda) & \to K \\
 970     (T, S) & \mapsto \operatorname{Tr}(T S)
 971   \end{align*}
 972 
 973   Indeed, \(\operatorname{rank} B_\lambda \le
 974   \operatorname{rank}(\mathcal{U}(\mathfrak{g})_0 \to
 975   \operatorname{End}(\mathcal{M}_\lambda)) \le \dim
 976   \operatorname{End}(\mathcal{M}_\lambda) = d^2\). Furthermore, if
 977   \(\operatorname{rank} B_\lambda = d^2\) then we must have
 978   \(\operatorname{rank}(\mathcal{U}(\mathfrak{g})_0 \to
 979   \operatorname{End}(\mathcal{M}_\lambda)) = d^2\) -- i.e. the map
 980   \(\mathcal{U}(\mathfrak{g})_0 \to \operatorname{End}(\mathcal{M}_\lambda)\)
 981   is surjective. In particular, if \(\operatorname{rank} B_\lambda = d^2\) then
 982   \(\mathcal{M}_\lambda\) is a simple \(\mathcal{U}(\mathfrak{g})_0\)-module,
 983   for if \(M \subset \mathcal{M}_\lambda\) is invariant under the action of
 984   \(\mathcal{U}(\mathfrak{g})_0\) then \(M\) is invariant under any
 985   \(K\)-linear operator \(\mathcal{M}_\lambda \to \mathcal{M}_\lambda\), so
 986   that \(M = 0\) or \(M = \mathcal{M}_\lambda\).
 987 
 988   On the other hand, if \(\mathcal{M}_\lambda\) is simple then by Burnside's
 989   Theorem on matrix algebras the map \(\mathcal{U}(\mathfrak{g})_0 \to
 990   \operatorname{End}(\mathcal{M}_\lambda)\) is surjective. Hence the
 991   commutativity of the previously drawn diagram, as well as the fact that
 992   \(\operatorname{rank}(\mathcal{U}(\mathfrak{g})_0 \to
 993   \operatorname{End}(\mathcal{M}_\lambda)) =
 994   \operatorname{rank}(\operatorname{End}(\mathcal{M}_\lambda)^* \to
 995   \mathcal{U}(\mathfrak{g})_0^*)\), imply that \(\operatorname{rank} B_\lambda
 996   = d^2\). This goes to show that \(U\) is precisely the set of all \(\lambda\)
 997   such that \(B_\lambda\) has maximal rank \(d^2\). We now show that \(U\) is
 998   Zariski-open. First, notice that
 999   \[
1000     U =
1001     \bigcup_{\substack{V \subset \mathcal{U}(\mathfrak{g})_0 \\ \dim V = d}}
1002     U_V,
1003   \]
1004   where \(U_V = \{\lambda \in \mathfrak{h}^* : \operatorname{rank}
1005   B_\lambda\!\restriction_V = d^2 \}\). Here \(V\) ranges over all
1006   \(d\)-dimensional subspaces of \(\mathcal{U}(\mathfrak{g})_0\) -- \(V\) is
1007   not necessarily a \(\mathcal{U}(\mathfrak{g})_0\)-submodule.
1008 
1009   Indeed, if \(\operatorname{rank} B_\lambda = d^2\) it follows from the
1010   subjectivity of the map \(\mathcal{U}(\mathfrak{g})_0 \to
1011   \operatorname{End}(\mathcal{M}_\lambda)\) that there is some \(V \subset
1012   \mathcal{U}(\mathfrak{g})_0\) with \(\dim V = d\) such that the restriction
1013   \(V \to \operatorname{End}(\mathcal{M}_\lambda)\) is surjective. The
1014   commutativity of
1015   \begin{center}
1016     \begin{tikzcd}
1017       V                                         \rar \dar  & V^* \\
1018       \operatorname{End}(\mathcal{M}_\lambda)   \rar{\sim} &
1019       \operatorname{End}(\mathcal{M}_\lambda)^* \uar
1020     \end{tikzcd}
1021   \end{center}
1022   then implies \(\operatorname{rank} B_\lambda\!\restriction_V = d^2\). In
1023   other words, \(U \subset \bigcup_V U_V\).
1024 
1025   Likewise, if \(\operatorname{rank} B_\lambda\!\restriction_V = d^2\) for some
1026   \(V\), then the commutativity of
1027   \begin{center}
1028     \begin{tikzcd}
1029       V                             \rar \dar & V^* \\
1030       \mathcal{U}(\mathfrak{g})_0   \rar      &
1031       \mathcal{U}(\mathfrak{g})_0^* \uar
1032     \end{tikzcd}
1033   \end{center}
1034   implies \(\operatorname{rank} B_\lambda \ge d^2\), which goes to show
1035   \(\bigcup_V U_V \subset U\).
1036 
1037   Given \(\lambda \in U_V\), the surjectivity of \(V \to
1038   \operatorname{End}(\mathcal{M}_\lambda)\) and the fact that \(\dim V <
1039   \infty\) imply \(V \to V^*\) is invertible. Since \(\mathcal{M}\) is a
1040   coherent family, \(B_\lambda\) depends polynomially in \(\lambda\). Hence so
1041   does the induced maps \(V \to V^*\). In particular, there is some Zariski
1042   neighborhood \(U'\) of \(\lambda\) such that the map \(V \to V^*\) induced by
1043   \(B_\mu\!\restriction_V\) is invertible for all \(\mu \in U'\).
1044 
1045   But the surjectivity of the map induced by \(B_\mu\!\restriction_V\) implies
1046   \(\operatorname{rank} B_\mu = d^2\), so \(\mu \in U_V\) and therefore \(U'
1047   \subset U_V\). This implies \(U_V\) is open for all \(V\). Finally, \(U\) is
1048   the union of Zariski-open subsets and is therefore open. We are done.
1049 \end{proof}
1050 
1051 The major remaining question for us to tackle is that of the existence of
1052 coherent extensions, which will be the focus of our next section.
1053 
1054 \section{Localizations \& the Existence of Coherent Extensions}
1055 
1056 Let \(M\) be a simple bounded \(\mathfrak{g}\)-module of degree \(d\). Our
1057 goal is to prove that \(M\) has a (unique) irreducible semisimple coherent
1058 extension \(\mathcal{M}\). Since \(M\) is simple, we know \(M \subset
1059 \mathcal{M}[\lambda]\) for any \(\lambda \in \operatorname{supp} M\). Our first
1060 task is constructing \(\mathcal{M}[\lambda]\). The issue here is that
1061 \(\operatorname{supp}_{\operatorname{ess}} M\) may not be all of \(\lambda + Q
1062 = \operatorname{supp}_{\operatorname{ess}} \mathcal{M}[\lambda]\), so we may
1063 find \(M \subsetneq \mathcal{M}[\lambda]\). In fact, we may find
1064 \(\operatorname{supp} M \subsetneq \lambda + Q\).
1065 
1066 This wasn't an issue an Example~\ref{ex:laurent-polynomial-mod} because we
1067 verified that the action of \(f \in \mathfrak{sl}_2(K)\) on \(K[x, x^{-1}]\) is
1068 injective. Since all weight spaces of \(K[x, x^{-1}]\) are \(1\)-dimensional,
1069 this implies the action of \(f\) is actually bijective, so we can obtain a
1070 nonzero vector in \(K[x, x^{-1}]_{2 k} = K x^k\) for any \(k \in \mathbb{Z}\)
1071 by translating between weight spaced using \(f\) and \(f^{-1}\) -- here
1072 \(f^{-1}\) denotes the \(K\)-linear operator \((-
1073 \sfrac{\mathrm{d}}{\mathrm{d}x} + \sfrac{x^{-1}}{2})^{-1}\), which is the
1074 inverse of the action of \(f\) on \(K[x, x^{-1}]\).
1075 \begin{center}
1076   \begin{tikzcd}
1077     \cdots     \rar[bend left=60]{f^{-1}}
1078     & K x^{-2} \rar[bend left=60]{f^{-1}} \lar[bend left=60]{f}
1079     & K x^{-1} \rar[bend left=60]{f^{-1}} \lar[bend left=60]{f}
1080     & K        \rar[bend left=60]{f^{-1}} \lar[bend left=60]{f}
1081     & K x      \rar[bend left=60]{f^{-1}} \lar[bend left=60]{f}
1082     & K x^2    \rar[bend left=60]{f^{-1}} \lar[bend left=60]{f}
1083     & \cdots   \lar[bend left=60]{f}
1084   \end{tikzcd}
1085 \end{center}
1086 
1087 In the general case, the action of some \(F_\alpha \in \mathfrak{g}\) with
1088 \(\alpha \in \Delta\) in \(M\) may not be injective. In fact, we have seen that
1089 the action of \(F_\alpha\) is injective for all \(\alpha \in \Delta^+\) if, and
1090 only if \(M\) is cuspidal. Nevertheless, we could intuitively \emph{make it
1091 injective} by formally inverting the elements \(F_\alpha \in
1092 \mathcal{U}(\mathfrak{g})\). This would allow us to obtain nonzero vectors in
1093 \(M_\mu\) for all \(\mu \in \lambda + Q\) by successively applying elements of
1094 \(\{F_\alpha^{\pm 1}\}_{\alpha \in \Delta}\) to a nonzero weight vector \(m \in
1095 M_\lambda\). Moreover, if the actions of the \(F_\alpha\) were to be
1096 invertible, we would find that all \(M_\mu\) are \(d\)-dimensional for \(\mu
1097 \in \lambda + Q\).
1098 
1099 In a commutative domain, this can be achieved by tensoring our module by the
1100 field of fractions. However, \(\mathcal{U}(\mathfrak{g})\) is hardly ever
1101 commutative -- \(\mathcal{U}(\mathfrak{g})\) is commutative if, and only if
1102 \(\mathfrak{g}\) is Abelian -- and the situation is more delicate in the
1103 non-commutative case. For starters, a non-commutative \(K\)-algebra \(A\) may
1104 not even have a ``field of fractions'' -- i.e. an over-ring where all elements
1105 of \(A\) have inverses. Nevertheless, it is possible to formally invert
1106 elements of certain subsets of \(A\) via a process known as
1107 \emph{localization}, which we now describe.
1108 
1109 \begin{definition}\index{localization!multiplicative subsets}\index{localization!Ore's condition}
1110   Let \(A\) be a \(K\)-algebra. A subset \(S \subset A\) is called
1111   \emph{multiplicative} if \(s \cdot t \in S\) for all \(s, t \in S\) and \(0
1112   \notin S\). A multiplicative subset \(S\) is said to satisfy \emph{Ore's
1113   localization condition} if for each \(a \in A\) and \(s \in S\) there exists
1114   \(b, c \in A\) and \(t, t' \in S\) such that \(s a = b t\) and \(a s = t'
1115   c\).
1116 \end{definition}
1117 
1118 \begin{theorem}[Ore-Asano]\index{localization!Ore-Asano Theorem}
1119   Let \(S \subset A\) be a multiplicative subset satisfying Ore's localization
1120   condition. Then there exists a (unique) \(K\)-algebra \(S^{-1} A\), with a
1121   canonical algebra homomorphism \(A \to S^{-1} A\), enjoying the universal
1122   property that each algebra homomorphism \(f : A \to B\) such that \(f(s)\) is
1123   invertible for all \(s \in S\) can be uniquely extended to an algebra
1124   homomorphism \(S^{-1} A \to B\). \(S^{-1} A\) is called \emph{the
1125   localization of \(A\) by \(S\)}, and the map \(A \to S^{-1} A\) is called
1126   \emph{the localization map}.
1127   \begin{center}
1128     \begin{tikzcd}
1129       A        \dar \rar{f}        & B \\
1130       S^{-1} A \urar[swap, dotted] &
1131     \end{tikzcd}
1132   \end{center}
1133 \end{theorem}
1134 
1135 If we identify an element with its image under the localization map, it follows
1136 directly from Ore's construction that every element of \(S^{-1} A\) has the
1137 form \(s^{-1} a\) for some \(s \in S\) and \(a \in A\). Likewise, any element
1138 of \(S^{-1} A\) can also be written as \(b t^{-1}\) for some \(t \in S\), \(b
1139 \in A\).
1140 
1141 Ore's localization condition may seem a bit arbitrary at first, but a more
1142 thorough investigation reveals the intuition behind it. The issue in question
1143 here is that in the non-commutative case we can no longer take the existence of
1144 common denominators for granted. However, the existence of common denominators
1145 is fundamental to the proof of the fact the field of fractions is a ring -- it
1146 is used, for example, to define the sum of two elements in the field of
1147 fractions. We thus need to impose their existence for us to have any hope of
1148 defining consistent arithmetics in the localization of an algebra, and Ore's
1149 condition is actually equivalent to the existence of common denominators --
1150 see the discussion in the introduction of \cite[ch.~6]{goodearl-warfield} for
1151 further details.
1152 
1153 We should also point out that there are numerous other conditions -- which may
1154 be easier to check than Ore's -- known to imply Ore's condition. For
1155 instance\dots
1156 
1157 \begin{lemma}
1158   Let \(S \subset A\) be a multiplicative subset generated by finitely many
1159   locally \(\operatorname{ad}\)-nilpotent elements -- i.e. elements \(s \in S\)
1160   such that for each \(a \in A\) there exists \(r > 0\) such that
1161   \(\operatorname{ad}(s)^r a = [s, [s, \cdots [s, a]]\cdots] = 0\). Then \(S\)
1162   satisfies Ore's localization condition.
1163 \end{lemma}
1164 
1165 In our case, we are more interested in formally inverting the action of
1166 \(F_\alpha\) on \(M\) than in inverting \(F_\alpha\) itself. To that end, we
1167 introduce one further construction, known as \emph{the localization of a
1168 module}.
1169 
1170 \begin{definition}\index{localization!localization of modules}
1171   Let \(S \subset A\) be a multiplicative subset satisfying Ore's localization
1172   condition and \(M\) be an \(A\)-module. The \(S^{-1} A\)-module \(S^{-1} M =
1173   S^{-1} A \otimes_A M\) is called \emph{the localization of \(M\) by \(S\)},
1174   and the homomorphism of \(A\)-modules
1175   \begin{align*}
1176     M & \to     S^{-1} M    \\
1177     m & \mapsto 1 \otimes m
1178   \end{align*}
1179   is called \emph{the localization map of \(M\)}.
1180 \end{definition}
1181 
1182 Notice that the \(S^{-1} A\)-module \(S^{-1} M\) has the natural structure of
1183 an \(A\)-module, where the action of \(A\) is given by the localization map \(A
1184 \to S^{-1} A\).
1185 
1186 It is interesting to observe that, unlike in the case of the field of fractions
1187 of a commutative domain, in general the localization map \(A \to S^{-1} A\) --
1188 i.e. the map \(a \mapsto \frac{a}{1}\) -- may not be injective. For instance,
1189 if \(S\) contains a divisor of zero \(s\), its image under the localization map
1190 is invertible and therefore cannot be a divisor of zero in \(S^{-1} A\). In
1191 particular, if \(a \in A\) is nonzero and such that \(s a = 0\) or \(a s = 0\)
1192 then its image under the localization map has to be \(0\). However, the
1193 existence of divisors of zero in \(S\) turns out to be the only obstruction to
1194 the injectivity of the localization map, as shown in\dots
1195 
1196 \begin{lemma}
1197   Let \(S \subset A\) be a multiplicative subset satisfying Ore's localization
1198   condition and \(M\) be an \(A\)-module. If \(S\) acts injectively on \(M\)
1199   then the localization map \(M \to S^{-1} M\) is injective. In particular, if
1200   \(S\) has no zero divisors then \(A\) is a subalgebra of \(S^{-1} A\).
1201 \end{lemma}
1202 
1203 Again, in our case we are interested in inverting the actions of the
1204 \(F_\alpha\) on \(M\). However, for us to be able to translate between all
1205 weight spaces associated with elements of \(\lambda + Q\), \(\lambda \in
1206 \operatorname{supp} M\), we only need to invert the \(F_\alpha\)'s for
1207 \(\alpha\) in some subset of \(\Delta\) which spans all of \(Q = \mathbb{Z}
1208 \Delta\). In other words, it suffices to invert \(F_\beta\) for all \(\beta\)
1209 in some basis \(\Sigma\) for \(\Delta\). We can choose such a basis to be
1210 well-behaved. For example, we can show\dots
1211 
1212 \begin{lemma}\label{thm:nice-basis-for-inversion}
1213   Let \(M\) be a simple infinite-dimensional bounded \(\mathfrak{g}\)-module.
1214   There is a basis \(\Sigma = \{\beta_1, \ldots, \beta_r\}\) for \(\Delta\)
1215   such that the elements \(F_{\beta_i}\) all act injectively on \(M\) and
1216   satisfy \([F_{\beta_i}, F_{\beta_j}] = 0\).
1217 \end{lemma}
1218 
1219 \begin{note}
1220   The basis \(\Sigma\) in Lemma~\ref{thm:nice-basis-for-inversion} may very
1221   well depend on the representation \(M\)! This is another obstruction to the
1222   functoriality of our constructions.
1223 \end{note}
1224 
1225 The proof of the previous Lemma is quite technical and was deemed too tedious
1226 to be included in here. See Lemma 4.4 of \cite{mathieu} for a full proof. Since
1227 \(F_\alpha\) is locally \(\operatorname{ad}\)-nilpotent for all \(\alpha \in
1228 \Delta\), we can see\dots
1229 
1230 \begin{corollary}
1231   Let \(\Sigma\) be as in Lemma~\ref{thm:nice-basis-for-inversion} and
1232   \((F_\beta)_{\beta \in \Sigma} \subset \mathcal{U}(\mathfrak{g})\) be the
1233   multiplicative subset generated by the \(F_\beta\)'s. The \(K\)-algebra
1234   \(\Sigma^{-1} \mathcal{U}(\mathfrak{g}) = (F_\beta)_{\beta \in \Sigma}^{-1}
1235   \mathcal{U}(\mathfrak{g})\) is well defined. Moreover, if we denote by
1236   \(\Sigma^{-1} M\) the localization of \(M\) by \((F_\beta)_{\beta \in
1237   \Sigma}\), the localization map \(M \to \Sigma^{-1} M\) is injective.
1238 \end{corollary}
1239 
1240 From now on let \(\Sigma\) be some fixed basis for \(\Delta\) satisfying the
1241 hypothesis of Lemma~\ref{thm:nice-basis-for-inversion}. We now show that
1242 \(\Sigma^{-1} M\) is a weight \(\mathfrak{g}\)-module whose support is an
1243 entire \(Q\)-coset.
1244 
1245 \begin{proposition}\label{thm:irr-bounded-is-contained-in-nice-mod}
1246   The restriction of the localization \(\Sigma^{-1} M\) is a bounded
1247   \(\mathfrak{g}\)-module of degree \(d\) with \(\operatorname{supp}
1248   \Sigma^{-1} M = Q + \operatorname{supp} M\) and \(\dim \Sigma^{-1} M_\lambda
1249   = d\) for all \(\lambda \in \operatorname{supp} \Sigma^{-1} M\).
1250 \end{proposition}
1251 
1252 \begin{proof}
1253   Fix some \(\beta \in \Sigma\). We begin by showing that \(F_\beta\) and
1254   \(F_\beta^{-1}\) map the weight space \(\Sigma^{-1} M_\lambda\) to
1255   \(\Sigma^{-1} M_{\lambda - \beta}\) and \(\Sigma^{-1} M_{\lambda + \beta}\),
1256   respectively. Indeed, given \(m \in M_\lambda\) and \(H \in \mathfrak{h}\) we
1257   have
1258   \[
1259     H \cdot (F_\beta \cdot m)
1260     = ([H, F_\beta] + F_\beta H) \cdot m
1261     = F_\beta (-\beta(H) + H) \cdot m
1262     = (\lambda - \beta)(H) F_\beta \cdot m
1263   \]
1264 
1265   On the other hand,
1266   \[
1267     0
1268     = [H, 1]
1269     = [H, F_\beta F_\beta^{-1}]
1270     = F_\beta [H, F_\beta^{-1}] + [H, F_\beta] F_\beta^{-1}
1271     = F_\beta [H, F_\beta^{-1}] - \beta(H) F_\beta F_\beta^{-1},
1272   \]
1273   so that \([H, F_\beta^{-1}] = \beta(H) \cdot F_\beta^{-1}\) and therefore
1274   \[
1275     H \cdot (F_\beta^{-1} \cdot m)
1276     = ([H, F_\beta^{-1}] + F_\beta^{-1} H) \cdot m
1277     = F_\beta^{-1} (\beta(H) + H) \cdot m
1278     = (\lambda + \beta)(H) F_\beta^{-1} \cdot m
1279   \]
1280 
1281   From the fact that \(F_\beta^{\pm 1}\) maps \(M_\lambda\) to \(\Sigma^{-1}
1282   M_{\lambda \pm \beta}\) follows our first conclusion: since \(M\) is a weight
1283   module and every element of \(\Sigma^{-1} M\) has the form \(s^{-1} \cdot m =
1284   s^{-1} \otimes m\) for \(s \in (F_\beta)_{\beta \in \Sigma}\) and \(m \in
1285   M\), we can see that \(\Sigma^{-1} M = \bigoplus_\lambda \Sigma^{-1}
1286   M_\lambda\). Furthermore, since the action of each \(F_\beta\) on
1287   \(\Sigma^{-1} M\) is bijective and \(\Sigma\) is a basis for \(Q\) we obtain
1288   \(\operatorname{supp} \Sigma^{-1} M = Q + \operatorname{supp} M\).
1289 
1290   Again, because of the bijectivity of the \(F_\beta\)'s, to see that \(\dim
1291   \Sigma^{-1} M_\lambda = d\) for all \(\lambda \in \operatorname{supp}
1292   \Sigma^{-1} M\) it suffices to show that \(\dim \Sigma^{-1} M_\lambda = d\)
1293   for some \(\lambda \in \operatorname{supp} \Sigma^{-1} M\). We may take
1294   \(\lambda \in \operatorname{supp} M\) with \(\dim M_\lambda = d\). For any
1295   finite-dimensional subspace \(V \subset \Sigma^{-1} M_\lambda\) we can find
1296   \(s \in (F_\beta)_{\beta \in \Sigma}\) such that \(s \cdot V \subset M\). If
1297   \(s = F_{\beta_{i_1}} \cdots F_{\beta_{i_r}}\), it is clear \(s \cdot V
1298   \subset M_{\lambda - \beta_{i_1} - \cdots - \beta_{i_r}}\), so \(\dim V =
1299   \dim s \cdot V \le d\). This holds for all finite-dimensional \(V \subset
1300   \Sigma^{-1} M_\lambda\), so \(\dim \Sigma^{-1} M_\lambda \le d\). It then
1301   follows from the fact that \(M_\lambda \subset \Sigma^{-1} M_\lambda\) that
1302   \(M_\lambda = \Sigma^{-1} M_\lambda\) and therefore \(\dim \Sigma^{-1}
1303   M_\lambda = d\).
1304 \end{proof}
1305 
1306 We now have a good candidate for a coherent extension of \(M\), but
1307 \(\Sigma^{-1} M\) is still not a coherent extension since its support is
1308 contained in a single \(Q\)-coset. In particular, \(\operatorname{supp}
1309 \Sigma^{-1} M \ne \mathfrak{h}^*\) and \(\Sigma^{-1} M\) is not a coherent
1310 family. To obtain a coherent family we thus need somehow extend \(\Sigma^{-1}
1311 M\). To that end, we will attempt to replicate the construction of the coherent
1312 extension of the \(\mathfrak{sl}_2(K)\)-module \(K[x, x^{-1}]\). Specifically,
1313 the idea is that if twist \(\Sigma^{-1} M\) by an automorphism which shifts its
1314 support by some \(\lambda \in \mathfrak{h}^*\), we can construct a coherent
1315 family by summing these modules over \(\lambda\) as in
1316 Example~\ref{ex:sl-laurent-family}.
1317 
1318 For \(K[x, x^{-1}]\) this was achieved by twisting the
1319 \(\operatorname{Diff}(K[x, x^{-1}])\)-module \(K[x, x^{-1}]\) by the
1320 automorphisms \(\varphi_\lambda : \operatorname{Diff}(K[x, x^{-1}]) \to
1321 \operatorname{Diff}(K[x, x^{-1}])\) and restricting the results to
1322 \(\mathcal{U}(\mathfrak{sl}_2(K))\) via the map
1323 \(\mathcal{U}(\mathfrak{sl}_2(K)) \to \operatorname{Diff}(K[x, x^{-1}])\), but
1324 this approach is inflexible since not every \(\mathfrak{sl}_2(K)\)-module
1325 factors through \(\operatorname{Diff}(K[x, x^{-1}])\). Nevertheless, we could
1326 just as well twist \(K[x, x^{-1}]\) by automorphisms of
1327 \(\mathcal{U}(\mathfrak{sl}_2(K))_f\) directly -- where
1328 \(\mathcal{U}(\mathfrak{sl}_2(K))_f = (f)^{-1} \mathcal{U}(\mathfrak{g})\) is
1329 the localization of \(\mathcal{U}(\mathfrak{sl}_2(K))\) by the multiplicative
1330 subset generated by \(f\).
1331 
1332 In general, we may twist the \(\Sigma^{-1} \mathcal{U}(\mathfrak{g})\)-module
1333 \(\Sigma^{-1} M\) by automorphisms of \(\Sigma^{-1}
1334 \mathcal{U}(\mathfrak{g})\). For \(\lambda = \beta \in \Sigma\) the map
1335 \begin{align*}
1336   \theta_\beta : \Sigma^{-1} \mathcal{U}(\mathfrak{g}) & \to
1337                  \Sigma^{-1} \mathcal{U}(\mathfrak{g}) \\
1338                  u & \mapsto F_\beta u F_\beta^{-1}
1339 \end{align*}
1340 is a natural candidate for such a twisting automorphism. Indeed, we will soon
1341 see that \(\twisted{(\Sigma^{-1} M)}{\theta_\beta}_\lambda = \Sigma^{-1}
1342 M_{\lambda + \beta}\). However, this is hardly useful to us, since \(\beta \in
1343 Q\) and therefore \(\beta + \operatorname{supp} \Sigma^{-1} M =
1344 \operatorname{supp} \Sigma^{-1} M\). If we want to expand the support of
1345 \(\Sigma^{-1} M\) we will have to twist by automorphisms that shift its support
1346 by \(\lambda \in \mathfrak{h}^*\) lying \emph{outside} of \(Q\).
1347 
1348 The situation is much less obvious in this case. Nevertheless, it turns out we
1349 can extend the family \(\{\theta_\beta\}_{\beta \in \Sigma}\) to a family of
1350 automorphisms \(\{\theta_\lambda\}_{\lambda \in \mathfrak{h}^*}\).
1351 Explicitly\dots
1352 
1353 \begin{proposition}\label{thm:nice-automorphisms-exist}
1354   There is a family of automorphisms \(\{\theta_\lambda : \Sigma^{-1}
1355   \mathcal{U}(\mathfrak{g}) \to \Sigma^{-1}
1356   \mathcal{U}(\mathfrak{g})\}_{\lambda \in \mathfrak{h}^*}\) such that
1357   \begin{enumerate}
1358     \item \(\theta_{k_1 \beta_1 + \cdots + k_r \beta_r}(u) = F_{\beta_1}^{k_1}
1359       \cdots F_{\beta_r}^{k_r} u F_{\beta_r}^{- k_r} \cdots F_{\beta_1}^{-
1360       k_1}\) for all \(u \in \Sigma^{-1} \mathcal{U}(\mathfrak{g})\) and \(k_1,
1361       \ldots, k_r \in \mathbb{Z}\).
1362 
1363     \item For each \(u \in \Sigma^{-1} \mathcal{U}(\mathfrak{g})\) the map
1364       \begin{align*}
1365         \mathfrak{h}^* & \to     \Sigma^{-1} \mathcal{U}(\mathfrak{g}) \\
1366                \lambda & \mapsto \theta_\lambda(u)
1367       \end{align*}
1368       is polynomial.
1369 
1370     \item If \(\lambda, \mu \in \mathfrak{h}^*\), \(N\) is a \(\Sigma^{-1}
1371       \mathcal{U}(\mathfrak{g})\)-module whose restriction to
1372       \(\mathcal{U}(\mathfrak{g})\) is a weight \(\mathfrak{g}\)-module and
1373       \(\twisted{N}{\theta_\lambda}\) is the \(\Sigma^{-1}
1374       \mathcal{U}(\mathfrak{g})\)-module \(N\) twisted by the automorphism
1375       \(\theta_\lambda\) then \(N_\mu = \twisted{N}{\theta_\lambda}_{\mu +
1376       \lambda}\). In particular, \(\operatorname{supp}
1377       \twisted{N}{\theta_\lambda} = \lambda + \operatorname{supp} N\).
1378   \end{enumerate}
1379 \end{proposition}
1380 
1381 \begin{proof}
1382   Since the elements \(F_\beta\), \(\beta \in \Sigma\) commute with one
1383   another, the endomorphisms
1384   \begin{align*}
1385     \theta_{k_1 \beta_1 + \cdots + k_r \beta_r}
1386     : \Sigma^{-1} \mathcal{U}(\mathfrak{g}) &
1387     \to \Sigma^{-1} \mathcal{U}(\mathfrak{g}) \\
1388     u & \mapsto
1389     F_{\beta_1}^{k_1} \cdots F_{\beta_r}^{k_r}
1390     u
1391     F_{\beta_1}^{- k_r} \cdots F_{\beta_r}^{- k_1}
1392   \end{align*}
1393   are well defined for all \(k_1, \ldots, k_r \in \mathbb{Z}\).
1394 
1395   Fix some \(u \in \Sigma^{-1} \mathcal{U}(\mathfrak{g})\). For any \(s \in
1396   (F_\beta)_{\beta \in \Sigma}\) and \(k > 0\) we have \(s^k u = \binom{k}{0}
1397   \operatorname{ad}(s)^0 u s^{k - 0} + \cdots + \binom{k}{k}
1398   \operatorname{ad}(s)^k u s^{k - k}\). Now if we take \(\ell\) such
1399   \(\operatorname{ad}(F_\beta)^{\ell + 1} u = 0\) for all \(\beta \in \Sigma\)
1400   we find
1401   \[
1402     \theta_{k_1 \beta_1 + \cdots + k_r \beta_r}(u)
1403     = \sum_{i_1, \ldots, i_r = 1, \ldots, \ell}
1404     \binom{k_1}{i_1} \cdots \binom{k_r}{i_r}
1405     \operatorname{ad}(F_{\beta_1})^{i_1} \cdots
1406     \operatorname{ad}(F_{\beta_r})^{i_r}
1407     u
1408     F_{\beta_1}^{- i_1} \cdots F_{\beta_r}^{- i_r}
1409   \]
1410   for all \(k_1, \ldots, k_r \in \mathbb{N}\).
1411 
1412   Since the binomial coefficients \(\binom{x}{k} = \frac{x (x-1) \cdots (x - k
1413   + 1)}{k!}\) can be uniquely extended to polynomial functions in \(x \in K\),
1414   we may in general define
1415   \[
1416     \theta_\lambda(u)
1417     = \sum_{i_1, \ldots, i_r \ge 0}
1418     \binom{\lambda_1}{i_1} \cdots \binom{\lambda_r}{i_r}
1419     \operatorname{ad}(F_{\beta_1})^{i_1} \cdots
1420     \operatorname{ad}(F_{\beta_r})^{i_r}
1421     r
1422     F_{\beta_1}^{- i_1} \cdots F_{\beta_r}^{- i_r}
1423   \]
1424   for \(\lambda_1, \ldots, \lambda_r \in K\), \(\lambda = \lambda_1 \beta_1 +
1425   \cdots + \lambda_r \beta_r \in \mathfrak{h}^*\).
1426 
1427   It is clear that the \(\theta_\lambda\) are endomorphisms. To see that the
1428   \(\theta_\lambda\) are indeed automorphisms, notice \(\theta_{- k_1 \beta_1 -
1429   \cdots - k_r \beta_r} = \theta_{k_1 \beta_1 + \cdots + k_r \beta_r}^{-1}\).
1430   The uniqueness of the polynomial extensions then implies \(\theta_{- \lambda}
1431   = \theta_\lambda^{-1}\) in general: given \(u \in \Sigma^{-1}
1432   \mathcal{U}(\mathfrak{g})\), the map
1433   \begin{align*}
1434     \mathfrak{h}^* & \to \Sigma^{-1} \mathcal{U}(\mathfrak{g})        \\
1435            \lambda & \mapsto \theta_\lambda(\theta_{-\lambda}(u)) - u
1436   \end{align*}
1437   is a polynomial extension of the zero map \(\mathbb{Z} \beta_1 \oplus \cdots
1438   \oplus \mathbb{Z} \beta_r \to \Sigma^{-1} \mathcal{U}(\mathfrak{g})\) and is
1439   therefore identically zero.
1440 
1441   Finally, let \(N\) be a \(\Sigma^{-1} \mathcal{U}(\mathfrak{g})\)-module
1442   whose restriction is a weight module. If \(n \in N\) then
1443   \[
1444     n \in \twisted{N}{\theta_\lambda}_{\mu + \lambda}
1445     \iff \theta_\lambda(H) \cdot n = (\mu + \lambda)(H) n
1446     \, \forall H \in \mathfrak{h}
1447   \]
1448 
1449   But
1450   \[
1451     \theta_\beta(H)
1452     = F_\beta H F_\beta^{-1}
1453     = ([F_\beta, H] + H F_\beta) F_\beta^{-1}
1454     = (\beta(H) + H) F_\beta F_\beta^{-1}
1455     = \beta(H) + H
1456   \]
1457   for all \(H \in \mathfrak{h}\) and \(\beta \in \Sigma\). In general,
1458   \(\theta_\lambda(H) = \lambda(H) + H\) for all \(\lambda \in \mathfrak{h}^*\)
1459   and hence
1460   \[
1461     \begin{split}
1462       n \in \twisted{N}{\theta_\lambda}_{\mu + \lambda}
1463       & \iff (\lambda(H) + H) \cdot n = (\mu + \lambda)(H) n
1464         \; \forall H \in \mathfrak{h} \\
1465       & \iff H \cdot n = \mu(H) n \; \forall H \in \mathfrak{h} \\
1466       & \iff n \in N_\mu
1467     \end{split},
1468   \]
1469   so that \(\twisted{N}{\theta_\lambda}_{\mu + \lambda} = N_\mu\).
1470 \end{proof}
1471 
1472 It should now be obvious\dots
1473 
1474 \begin{proposition}[Mathieu]\label{thm:coh-ext-exists}
1475   There exists a coherent extension \(\mathcal{M}\) of \(M\).
1476 \end{proposition}
1477 
1478 \begin{proof}
1479   Take\footnote{Here we fix some $\lambda_\xi \in \xi$ for each $Q$-coset $\xi
1480   \in \mfrac{\mathfrak{h}^*}{Q}$. While there is a natural isomorphism
1481   $\twisted{(\Sigma^{-1} M)}{\theta_\lambda} \isoto \twisted{(\Sigma^{-1}
1482   M)}{\theta_\mu}$ for each $\mu \in \lambda + Q$, they are not the same
1483   \(\mathfrak{g}\)-modules strictly speaking. This is yet another obstruction
1484   to the functoriality of our constructions.}
1485   \[
1486     \mathcal{M}
1487     = \bigoplus_{\lambda + Q \in \mfrac{\mathfrak{h}^*}{Q}}
1488       \twisted{(\Sigma^{-1} M)}{\theta_\lambda}
1489   \]
1490 
1491   It is clear \(M\) lies in \(\Sigma^{-1} M = \twisted{(\Sigma^{-1}
1492   M)}{\theta_0}\) and therefore \(M \subset \mathcal{M}\). On the other hand,
1493   \(\dim \mathcal{M}_\mu = \dim \twisted{(\Sigma^{-1} M)}{\theta_\lambda}_\mu =
1494   \dim \Sigma^{-1} M_{\mu - \lambda} = d\) for all \(\mu \in \lambda + Q\) --
1495   \(\lambda\) standing for some fixed representative of its \(Q\)-coset.
1496   Furthermore, given \(u \in \mathcal{U}(\mathfrak{g})_0\) and \(\mu \in
1497   \lambda + Q\),
1498   \[
1499     \operatorname{Tr}(u\!\restriction_{\mathcal{M}_\mu})
1500     = \operatorname{Tr}
1501       (\theta_\lambda(u)\!\restriction_{\Sigma^{-1} M_{\mu - \lambda}})
1502   \]
1503   is polynomial in \(\mu\) because of the second item of
1504   Proposition~\ref{thm:nice-automorphisms-exist}.
1505 \end{proof}
1506 
1507 Lo and behold\dots
1508 
1509 \begin{theorem}[Mathieu]\label{thm:mathieu-ext-exists-unique}\index{coherent family!Mathieu's \(\mExt\) coherent extension}
1510   There exists a unique semisimple coherent extension \(\mExt(M)\) of \(M\).
1511   More precisely, if \(\mathcal{M}\) is any coherent extension of \(M\), then
1512   \(\mathcal{M}^{\operatorname{ss}} \cong \mExt(M)\). Furthermore, \(\mExt(M)\)
1513   is an irreducible coherent family.
1514 \end{theorem}
1515 
1516 \begin{proof}
1517   The existence part should be clear from the previous discussion: it suffices
1518   to fix some coherent extension \(\mathcal{M}\) of \(M\) and take
1519   \(\mExt(M) = \mathcal{M}^{\operatorname{ss}}\).
1520 
1521   To see that \(\mExt(M)\) is irreducible, recall from
1522   Corollary~\ref{thm:bounded-is-submod-of-extension} that \(M\) is a
1523   \(\mathfrak{g}\)-submodule of \(\mExt(M)\). Since the degree of \(M\) is the
1524   same as the degree of \(\mExt(M)\), some of its weight spaces have maximal
1525   dimension inside of \(\mExt(M)\). In particular, it follows from
1526   Lemma~\ref{thm:centralizer-multiplicity} that \(\mExt(M)_\lambda =
1527   M_\lambda\) is a simple \(\mathcal{U}(\mathfrak{g})_0\)-module for some
1528   \(\lambda \in \operatorname{supp} M\).
1529 
1530   As for the uniqueness of \(\mExt(M)\), fix some other semisimple coherent
1531   extension \(\mathcal{N}\) of \(M\). We claim that the multiplicity of a given
1532   simple \(\mathfrak{g}\)-module \(L\) in \(\mathcal{N}\) is determined by its
1533   \emph{trace function}
1534   \begin{align*}
1535     \mathfrak{h}^* \times \mathcal{U}(\mathfrak{g})_0 &
1536     \to K \\
1537     (\lambda, u) &
1538     \mapsto \operatorname{Tr}(u\!\restriction_{\mathcal{N}_\lambda})
1539   \end{align*}
1540 
1541   It is a well known fact of the theory of modules that, given an associative
1542   \(K\)-algebra \(A\), a finite-dimensional semisimple \(A\)-module \(L\) is
1543   determined, up to isomorphism, by its \emph{character}
1544   \begin{align*}
1545     \chi_L : A & \to     K                                    \\
1546              a & \mapsto \operatorname{Tr}(a\!\restriction_L)
1547   \end{align*}
1548 
1549   In particular, the multiplicity of \(L\) in \(\mathcal{N}\), which is the
1550   same as the multiplicity of \(L_\lambda\) in \(\mathcal{N}_\lambda\), is
1551   determined by the character \(\chi_{\mathcal{N}_\lambda} :
1552   \mathcal{U}(\mathfrak{g})_0 \to K\). Since this holds for all simple weight
1553   \(\mathfrak{g}\)-modules, it follows that \(\mathcal{N}\) is determined by
1554   its trace function. Of course, the same holds for \(\mExt(M)\). We now claim
1555   that the trace function of \(\mathcal{N}\) is the same as that of
1556   \(\mExt(M)\). Clearly,
1557   \(\operatorname{Tr}(u\!\restriction_{\mExt(M)_\lambda}) =
1558   \operatorname{Tr}(u\!\restriction_{M_\lambda}) =
1559   \operatorname{Tr}(u\!\restriction_{\mathcal{N}_\lambda})\) for all \(\lambda
1560   \in \operatorname{supp}_{\operatorname{ess}} M\), \(u \in
1561   \mathcal{U}(\mathfrak{g})_0\). Since the essential support of \(M\) is
1562   Zariski-dense and the maps \(\lambda \mapsto
1563   \operatorname{Tr}(u\!\restriction_{\mExt(M)_\lambda})\) and \(\lambda \mapsto
1564   \operatorname{Tr}(u\!\restriction_{\mathcal{N}_\lambda})\) are polynomial in
1565   \(\lambda \in \mathfrak{h}^*\), it follows that these maps coincide for all
1566   \(u\).
1567 
1568   In conclusion, \(\mathcal{N} \cong \mExt(M)\) and \(\mExt(M)\) is unique.
1569 \end{proof}
1570 
1571 A sort of ``reciprocal'' of Theorem~\ref{thm:mathieu-ext-exists-unique} also
1572 holds. Namely\dots
1573 
1574 \begin{proposition}\label{thm:coherent-families-are-all-ext}
1575   Let \(\mathcal{M}\) be a semisimple irreducible coherent family and \(M
1576   \subset \mathcal{M}\) be an infinite-dimensional simple submodule. Then
1577   \(\mathcal{M} \cong \mExt(M)\). In particular, all semisimple coherent
1578   families have the form \(\mathcal{M} \cong \mExt(M)\) for some simple bounded
1579   \(\mathfrak{g}\)-module \(M\).
1580 \end{proposition}
1581 
1582 \begin{proof}
1583   Since \(M \subset \mathcal{M}\), \(M\) is bounded and
1584   \(\operatorname{supp}_{\operatorname{ess}} M\) is Zariski-dense. In addition,
1585   it follows from Lemma~\ref{thm:set-of-simple-u0-mods-is-open} that \(U =
1586   \{\lambda \in \mathfrak{h}^* : \mathcal{M}_\lambda \ \text{is a simple
1587   $\mathcal{U}(\mathfrak{g})_0$-module}\}\) is a Zariski-open subset -- which
1588   is non-empty since \(\mathcal{M}\) is irreducible.
1589 
1590   Hence there is some \(\lambda \in \operatorname{supp}_{\operatorname{ess}} M
1591   \cap U\). In particular, there is some \(\lambda \in
1592   \operatorname{supp}_{\operatorname{ess}} M\) such that \(M_\lambda =
1593   \mathcal{M}_\lambda\) and thus \(\deg M = \dim \mathcal{M}_\lambda = \deg
1594   \mathcal{M}\). This implies that \(\mathcal{M}\) is a coherent extension of
1595   \(M\), so that by the uniqueness of semisimple irreducible coherent
1596   extensions we get \(\mathcal{M} \cong \mExt(M)\).
1597 \end{proof}
1598 
1599 Having thus reduced the problem of classifying the cuspidal
1600 \(\mathfrak{g}\)-modules to that of understanding semisimple irreducible
1601 coherent families, the only remaining question for us to tackle is: what are
1602 the coherent \(\mathfrak{g}\)-families? This turns out to be a decently
1603 complicated question on its own, and we will require a full chapter to answer
1604 it. This will be the focus of our final chapter.